1 Q&A Usmle Step 1
1 Q&A Usmle Step 1
1 Q&A Usmle Step 1
Cardiovascular
161
162 Section II: Organ Systems • Questions
Q u e st i o n s
cholesterol
(B) Decreased production of triglycerides
(C) Downregulation of cell surface LDL cho-
lesterol receptors
(D) Increased production of HDL cholesterol
(E) Upregulation of cell surface LDL choles-
terol receptors
5. A 2-year-old boy is brought to the clinic by vere. After a thorough work-up he is diagnosed
his parents because he suffers from sudden with an acute myocardial infarction (MI).
cyanotic attacks that can be improved only by Which of the following laboratory test eleva-
squatting. He is referred to a cardiologist, who tions is most specific for MI?
informs the parents that their son’s right ventri-
(A) Alanine aminotransferase
cle is abnormally large. Which of the following
(B) Aspartate aminotransferase
is most likely the root cause of this boy’s heart
(C) Creatine kinase-MB fraction
defect?
(D) Lactate dehydrogenase
(A) Anterosuperior displacement of the infun- (E) Troponin I
dibular septum
(B) Overriding aorta 8. Following the administration of drug X, there
(C) Patent ductus arteriosus is an increase in systolic, diastolic, and mean
High-Yield Systems
(D) Pulmonary stenosis arterial pressures. After the effect of drug
(E) Ventricular septal defect X has worn off completely, drug Y is then
added, resulting in little or no change to the
6. A 55-year-old man with hypertension is pre- baseline blood pressure. When drug X is re-
scribed an antiarrhythmic agent that alters the administered, there is a net decrease in blood
flow of cations in myocardial tissue. The im- pressure (see image). Which of the following
age is a trace of a myocardial action potential. drug combinations represents drug X and drug
Each phase is associated with the opening and/ Y, respectively?
or closing of various ion channels. Which of
the following would be affected by an agent
that affects phase 0 of the myocardial action X Y X
potential?
1
2 Reproduced, with permission, from USMLERx.com.
Cardiovascular
30 ms
(A) Epinephrine, phentolamine
0
3
(B) Isoproterenol, clonidine
(C) Norepinephrine, propranolol
(D) Phenylephrine, metoprolol
(E) Phenylephrine, phentolamine
Reproduced, with permission, from USMLERx.com. 9. A 56-year-old woman arrives in the emer-
gency department complaining of dizziness
and headache. Her blood pressure is 210/140
(A) Ligand-gated calcium channels opening mm Hg. She is currently not taking any med-
(B) Ligand-gated potassium channels closing ications and has not seen a doctor for several
(C) Ligand-gated sodium channels opening years. The physician decides to address her
(D) Voltage-gated calcium channels opening hypertension urgently. Which of the following
(E) Voltage-gated potassium channels closing drugs is contraindicated in this patient?
(F) Voltage-gated sodium channels closing
(G) Voltage-gated sodium channels opening (A) Intravenous diltiazem
(B) Intravenous labetalol
7. A 50-year-old man presents to the emergency (C) Intravenous metoprolol
department because of substernal chest pain (D) Oral captopril
that started four hours ago and is becoming se- (E) Sublingual nifedipine
164 Section II: Organ Systems • Questions
10. A 64-year-old man is brought to the emergency (D) Protein deposits such as transthyretin
department for chest pain. An electrocardio- (E) Trypanosome transmitted by insect bite
gram demonstrates an ST-elevation myocardial
infarction in the posterior leads. After the ini- 12. A 72-year-old African-American man under-
tial management, he is admitted to the critical goes hip surgery. On his third hospital day, he
care unit for monitoring. On the fifth hospital develops chest pain, tachycardia, dyspnea, and
day, he experiences a sudden onset of dyspnea a low-grade fever. The man goes into cardiac
and hypotension. An echocardiogram shows arrest and efforts to resuscitate him are unsuc-
severe mitral regurgitation. An occlusion in cessful. On autopsy a massive pulmonary em-
which vessel is responsible for these findings? bolus is discovered. Which of the following
would most likely predispose the patient to this
(A) Left anterior descending artery
event?
(B) Left circumflex artery
High-Yield Systems
(C) Pneumonitis
(D) Strawberry tongue
(E) Weak pulses in the upper extremities
High-Yield Systems
(E) Verapamil (D) Monocyte activation
(E) Plaque formation
15. A 65-year-old African-American man is ad-
mitted to the hospital for severe shortness of 18. Jugular venous pressure (JVP) curves are de-
breath. He states that he has been having in- signed to show the pressure changes that nor-
creased difficulty breathing when performing mally take place in the right atrium through-
physical activity. Lung auscultation reveals bi- out the cardiac cycle. A JVP curve consists of
lateral crackles. His blood pressure is 125/85 two, or sometimes three, positive waves and
mm Hg, and his heart rate is 85/min. Cardiac two negative troughs. A normal JVP curve is
auscultation detects no rubs or murmurs. shown in the image. Which of the following
Blood tests are not suggestive of infection. points on the normal jugular venous tracing
Coronary artery angiography is performed and below would be most prominently affected in
shows no significant vascular disease. What is tricuspid regurgitation?
the most likely cause of this patient’s disease
manifestations?
R
(A) Aortic stricture
Cardiovascular
(B) Fibrosis of the endocardium
(C) Fibrotic debris within the pericardial
P T
membrane ECG
(D) Ischemia of the myocardium tracing
Q
(E) Ventricular dysfunction
mm Hg
S
a c
16. A 25-year-old pregnant woman goes to her v
y
Jugular x
gynecologist for her 36-week checkup. She tracing
complains of light-headedness when she lies
down in bed at night. In the office her blood
pressure is 120/70 mm Hg while sitting upright Reproduced, with permission, from USMLERx.com.
and 90/50 mm Hg while lying supine. Which
of the following is the most likely cause of this
hypotension? (A) C and X
(B) A and Y
(A) Anemia (C) A and C
(B) Cardiogenic shock (D) V and Y
(C) Inferior vena cava compression
(D) Third spacing of fluid
(E) Vasodilation
166 Section II: Organ Systems • Questions
specific location?
disappeared. What is the mechanism of action
(A) Ascending colon of the drug most likely prescribed?
(B) Sigmoid colon
(C) Small intestine (A) Increases thromboxane A2
(D) Spleen (B) Inhibits prostaglandin E2 formation
(E) Stomach (C) Stimulates M2-receptors
(D) Stimulates prostaglandin E2-receptors
21.
A 52-year-old African-American man is (E) Stimulates prostaglandin F2a-receptors
brought to the emergency department unre-
sponsive. Efforts to resuscitate him are unsuc- 23. A 25-year-old white woman with no past medi-
cessful. On autopsy, they discover that he suf- cal history presents to the emergency depart-
fered from a ruptured aneurysm of the aortic ment for “a racing heartbeat.” It is determined
root. A picture of his dilated aorta is shown be- that she has paroxysmal supraventricular tachy-
low. In addition, inspection of the man’s skin cardia. Which of the following is the drug of
reveals several nodular lesions that are present choice used for diagnosing and abolishing
throughout his trunk and extremities. Which atrioventricular nodal arrhythmias by virtue of
of the following is most likely associated with its effectiveness and its low toxicity?
the underlying etiology of this patient’s aneu- (A) Adenosine
rysm? (B) Bretylium
Chapter 8: Cardiovascular • Questions 167
(C) Encainide lar volume for a single cardiac cycle. This pa-
(D) Lidocaine tient’s S4 heart sound heard on auscultation
(E) Sotalol would best correspond to which of the follow-
ing points?
24. Gemfibrozil has been proven to modestly
decrease LDL cholesterol levels, modestly
140
increase HDL cholesterol levels, and signifi- F
cantly decrease triglyceride levels. The mecha- 120 E
nism of gemfibrozil is best described by which D
100
of the following?
Pressure (mmHg)
80
(A) Slows the conversion of VLDL to LDL
cholesterol 60
High-Yield Systems
(B) Increases expression of triglyceride recep- 40
tors C
(C) Inhibits peroxisome proliferator-activated 20 B
receptor a nuclear transcription (PPAR-a) 0
regulator 40 85 130
Ventricular volume (mL)
(D) Promotes hepatocyte lipolysis A
(E) Activates lipoprotein lipase
Reproduced, with permission, from USMLERx.com.
25. A 48-year-old executive presents to the emer-
gency department because of chest tight-
(A) A
ness and shortness of breath. ECG shows ST-
(B) B
segment elevations in leads V4, V5, and V6.
(C) C
He has a history of high blood pressure, and
(D) D
his father died of heart problems at a young
(E) E
age. Assuming no other cardiac history, which
(F) F
of the following myocardial abnormalities
Cardiovascular
would most likely be seen via light microscopy
27. A 67-year-old woman with a long history
eight hours after his symptoms began?
of poorly controlled diabetes mellitus and
(A) Contraction bands chronic renal failure is admitted to the hospi-
(B) Granulation tissue tal for treatment of cellulitis. Two days into her
(C) Monocytic infiltrate hospital stay she complains of chest pain that
(D) Neutrophilic infiltrate is relieved when she leans forward. An ECG
(E) No change can be detected with light mi- shows diffuse ST segment elevations with PR
croscopy at this time depressions; her echocardiogram is normal.
Which of the following is the most appropriate
26. A 58-year-old man with a past medical his- treatment at this time?
tory of hypertension goes to his physician for
(A) Cardiac catheterization
a routine visit. On physical examination the
(B) Dialysis
physician is able to detect an S4 heart sound,
(C) Nonsteroidal anti-inflammatory drugs
and refers the patient to a cardiologist. After a
(D) Pericardiocentesis
thorough work-up, he is found to have left ven-
(E) Switch her to another antibiotic regimen
tricular hypertrophy. The image below plots
left ventricular pressure versus left ventricu-
168 Section II: Organ Systems • Questions
28. An 87-year-old man suffered an acute ST- 30. A 65-year-old woman with type 2 diabetes mel-
elevation myocardial infarction a few minutes litus is prescribed a drug by her physician to
ago. He was subsequently treated with aspirin, treat her hypertension. She returns complain-
metoprolol, and heparin. Immediately be- ing of facial swelling and a cough. Her physi-
fore being taken to the catheterization labora- cian promptly switches her to a new drug,
tory, the patient becomes unresponsive. He which has the same desired effect but fewer
is placed on telemetry and his rhythm strip is adverse effects. What is the mechanism of ac-
shown in the image. Although the patient re- tion of the replacement drug?
ceives a series of emergent defibrillations, he
(A) Blockade of b1 receptors
does not convert to sinus rhythm. Due to fail-
(B) Inhibition of angiotensin II receptors
ure to respond, the patient is given an agent as-
(C) Inhibition of angiotensin-converting en-
sociated with which of the following potential
zyme
High-Yield Systems
adverse effects?
(D) Inhibition of calcium channels
(E) Inhibition of NaCl reabsorption in the
early distal tubule
B
A C D
P P P P P P P P P P P P
1 2
Reproduced, with permission, from USMLERx.com. Pain Days
(A) Babesiosis
(B) Epidemic typhus (A) Aspartate aminotransferase
(C) Malaria (B) Brain natriuretic peptide
High-Yield Systems
(D) Plague (C) C-reactive protein
(E) Rocky Mountain spotted fever (D) Creatine kinase-MB fraction
(E) Lactate dehydrogenase
33. A 55-year-old woman with a history of myocar- (F) Myoglobin
dial infarction (MI) in the area of distribution (G) Troponin
of the left anterior descending artery (LAD)
presents to her cardiologist because of fatigue, 35. A girl is born prematurely at 26 weeks’ gesta-
orthopnea, nocturnal dyspnea, weight gain, tion. She is placed in an incubator and appears
and “swollen ankles.” Echocardiography re- to be in stable condition. During a cardiac ex-
veals normal timing of the cardiac cycle and amination on her second day of life, the physi-
normal cardiac valves. Which of the following cian hears a machine-like murmur in the sec-
would most likely be heard in this patient on ond intercostal space at the left sternal border.
auscultation? He prescribes indomethacin to eliminate the
(A) Holosystolic, harsh-sounding murmur, condition before complications arise. What is
loudest at tricuspid area the embryonic origin of the structure targeted
(B) Paradoxically split S2 by this drug?
Cardiovascular
(C) Pulmonary flow murmur (A) First aortic arch
(D) S3 (B) Fourth aortic arch
(E) S4 (C) Second aortic arch
(F) Widened splitting of S2 (D) Sixth aortic arch
(E) Third aortic arch
34. Elevations in cardiac enzymes are used to di-
agnose a myocardial infarction (MI). The
diagram below shows the duration of eleva-
tion of several of these enzymes following an
MI. Which cardiac enzyme becomes elevated
approximately four hours after an MI and re-
mains elevated 7-10 days afterward?
170 Section II: Organ Systems • Questions
36. A 61-year-old man is rushed to the emergency 38. A 2-year-old Japanese girl is brought to the
department after losing consciousness for a few clinic by her parents, who are concerned about
minutes at a shopping mall. His wife explained a persistent fever of 40°C (104°F) and an ery-
to the paramedics that he was walking up a thematous rash on her trunk that has been
flight of stairs when he became short of breath present for the past week. Laboratory tests
and fell to the ground. Once at the hospital, show an erythrocyte sedimentation rate of 80
the patient reports to the physician that over mm/hr, a C-reactive protein level of 15 mg/L,
the past three months, he has been feeling and a platelet count of 700,000/mm³. The
dizzy frequently, in addition to becoming short doctors decide to treat her with intravenous
of breath after walking five blocks. He reports immunoglobulin and aspirin to avoid a life-
that two years ago he was able to walk a mile threatening complication. What other signs
with no difficulty. On physical examination, might this patient’s doctors have seen that led
High-Yield Systems
the physician notes some crackles in the lung them to initiate this therapy?
bases bilaterally and carotid pulses that seem
(A) Hypothermia
somewhat weak and delayed in comparison to
(B) Optic neuritis
the patient’s heart sounds. Which other physi-
(C) Proteinuria
cal finding is likely to be found in this patient?
(D) Pulmonary edema
(A) Blowing holosystolic murmur that radiates (E) Strawberry tongue
to the axilla
(B) Crescendo-decrescendo systolic murmur 39. A 54-year-old woman comes to the physician
that radiates to the axilla three months after a undergoing a root canal,
(C) Crescendo-decrescendo systolic murmur with primary complaints of persistent general
that radiates to the carotids malaise and fever. The symptoms developed
(D) Diastolic decrescendo murmur that radi- slowly over the weeks following her root canal,
ates to the apex but have not abated. The patient has a temper-
(E) Diastolic decrescendo murmur that radi- ature of 38.3°C (100.1°F). Ophthalmic exami-
ates to the axilla nation reveals retinal hemorrhages with clear
(F) Opening snap followed by a mid-diastolic central regions. Examination of the extremi-
Cardiovascular
rumble that radiates to the axilla ties reveals painful red nodules on her dig-
its and dark macules on her palms and soles.
37. A 65-year-old man presents to the emergency On cardiac examination, a click and a systolic
department because of chest pain. He is found murmur are auscultated over the mitral valve.
to have a large inferior wall myocardial infarc- She tells the physician that the click is due to
tion with 2-mm ST-segment elevations in leads a mechanical valve replacement done four
II, III, and aVF. ECG also demonstrates grad- years ago because she had rheumatic fever as
ual lengthening of the PR interval until one a child. Which of the following is the most ap-
QRS complex is dropped. He is sent to the car- propriate treatment?
diac catheterization laboratory, where a stent is
(A) Acyclovir
placed in one of his major coronary arteries. In
(B) Caspofungin
which coronary artery was the stent most likely
(C) Clindamycin
placed?
(D) Mebendazole
(A) Diagonal branch of the LAD (E) Metronidazole
(B) Left anterior descending artery (F) Penicillin
(C) Left circumflex artery (G) Pentamidine
(D) Left marginal artery
(E) Right coronary artery
Chapter 8: Cardiovascular • Questions 171
Pressure, mm Hg
hours. Forty-five minutes earlier he had an-
nounced that he was going to the bedroom to
administer his nightly dose of insulin. When II
IV
he didn’t return to the living room, she went to
20
check on him, and found him lying face down
on the bed with an empty insulin needle be-
side him and an open bottle of propranolol on I
the nightstand. Upon arrival in the emergency
High-Yield Systems
department the patient is unresponsive. Heart
rate is 45/min and his serum glucose level is 80 40 80 120 160
mg/dL. Which of the following is the most ap- Volume, cc
propriate treatment to reverse the most likely
Reproduced, with permission, from USMLERx.com.
cause of this patient’s condition?
(A) Albuterol
(B) Cocaine (A) Phase I
(C) Digitalis (B) Phase II
(D) Glucagon (C) Phase III
(E) Phenylephrine (D) Phase IV
41. Nitrates often are given as part of the manage- 43. A 32-year-old man with diabetes presents to his
ment of angina. What is the primary mecha- physician with orthostatic hypotension. A defi-
nism by which nitrates work in the treatment ciency in the normal physiologic response car-
of this condition? ried out by arterial baroreceptors located in the
aortic arch and the carotid sinus is suspected.
(A) Decrease atrioventricular node conduction
Cardiovascular
What is the mechanism of the normal physi-
velocity ologic response to hypotension?
(B) Decrease heart rate and contractility
(C) Increase oxygen delivery to myocytes (A) Decreased baroreceptor afferent firing in
(D) Stimulate dilation of coronary arterioles the aortic arch leads to increased sympa-
(E) Stimulate venodilation thetic efferent firing
(B) Decreased baroreceptor afferent firing in
42. The image depicts the relationship of left ven- the carotid sinus leads to increased para-
tricular pressure and volume in the cardiac cy- sympathetic efferent firing
cle. The various phases of the cardiac cycle are (C) Decreased baroreceptor afferent firing in
labeled I through IV. During which phase are the carotid sinus leads to increased sympa-
the pressures in the left atrium and left ventri- thetic efferent firing
cle most equal? (D) Increased baroreceptor afferent firing in
the aortic arch leads to increased parasym-
pathetic efferent firing
(E) Increased baroreceptor afferent firing in
the carotid sinus leads to increased sympa-
thetic efferent firing
172 Section II: Organ Systems • Questions
of heart block?
(D) Thromboembolism
(E) Ventricular rupture (A) Atrioventricular node abnormality
(B) Defect in the His-Purkinje system
45.
A 45-year-old African-American man is (C) Independently contracting atria and ventri-
brought to the emergency department because cles
of sudden chest pain radiating to the back. The (D) Retrograde conduction
pain started while the patient was taking his (E) Sinoatrial node abnormality
morning jog. X-ray of the chest is immediately
done and shows a widened mediastinum. The 47. A 56-year-old white man is rushed to the emer-
patient has a history of anaphylaxis related to gency department after complaining of crush-
the type of iodinated contrast agent used in ing substernal chest pain. He is morbidly
CT. Therefore, an MRI is performed to con- obese, diaphoretic, tachypneic, and clutching
firm the expected diagnosis. Findings on a sag- his chest. Initial ECG reveals ST-segment el-
ittal bright blood MRI are shown in the image. evations in the anterior leads. The patient is
Which of the following co-morbidities is most stabilized and seems to be doing well, but then
directly responsible for the patient’s potentially he suddenly experiences cardiac arrest and
Cardiovascular
48. The image is a representation of the pressure- 49. Cardiac output is a function of stroke volume
volume (P-V) relationship in the left ventricle and heart rate. Stroke volume increases when
during a typical cardiac cycle. The phases contractility increases, preload increases, or af-
of the cardiac cycle are labeled I through IV. terload decreases. There are a number of fac-
Which of the following occurrences alone tors that affect each of these components and
would increase the width of the P-V loop? ultimately cardiac output. Which of the follow-
ing variations would increase cardiac output in
an otherwise normal patient?
(A) b-Blocker treatment
110 (B) Cardiac glycoside administration
III (C) Increasing extracellular sodium concentra-
Pressure, mm Hg
tion
High-Yield Systems
(D) Lowering intracellular calcium concentra-
tion
IV II
(E) Metabolic acidosis
20
Cardiovascular
(D) Increased arterial pressure
(E) Increased end-systolic volume
174 Section II: Organ Systems • Answers
An s w e r s
1. The correct answer is D. This patient has Answer A is incorrect. This is the mechanism
Wolff-Parkinson-White (WPW) syndrome, a of calcium channel blockers such as nifedi
condition whereby the heart is abnormally pine, not of hydralazine.
stimulated by an accessory electrical pathway
Answer B is incorrect. Hydralazine works by
(often known as the bundle of Kent). The
increasing cGMP, not decreasing it.
presence of Δ-wave (a sloping upstroke of the
QRS complex) is diagnostic. Amiodarone is Answer D is incorrect. This is the mechanism
an effective pharmacologic agent for this con- of angiotensin-converting enzyme (ACE) in-
dition. This is a potassium channel blocker hibitors, not of hydralazine.
(class III antiarrhythmic). Procainamide also
High-Yield Systems
ECG shows peak T waves and a widened Answer C is incorrect. Patent ductus arterio-
QRS interval, which are classic changes seen sus is not the cause of tetralogy of Fallot. In
in hyperkalemia. Spironolactone is the most fact, a patent ductus arteriosus is protective
likely medication to affect urinary electrolytes. in patients with tetralogy of Fallot because it
As an inhibitor of aldosterone receptors in causes some of the unoxygenated blood from
the collecting tubule and an inhibitor of Na+ the overriding aorta to return to the pulmonary
channels, spironolactone greatly decreases the artery to be oxygenated.
excretion of K+ and mildly increases the excre-
Answer D is incorrect. Pulmonary stenosis is
tion of Na+. Urine volume will be high-normal
one of the four manifestations of tetralogy of
because the diuretic will increase saltwater
Fallot, not the cause.
wasting.
Answer E is incorrect. VSD is one of the four
Answer A is incorrect. Spironolactone de-
High-Yield Systems
manifestations of tetralogy of Fallot, not the
creases K+ excretion, so there will be decreased
cause.
levels of K+ in the urine sample.
Answer B is incorrect. Na+ excretion will be 6. The correct answer is G. Voltage-gated Ca+
increased with the use of spironolactone; also, channels (L type) open slowly in response to
diuretics will increase the amount of urine vol- the sodium upstroke (approximately around
ume excreted. -40 mV), allowing calcium to flow down its
concentration gradient and into the cell. Con-
Answer C is incorrect. Spironolactone will
currently, there is an outward potassium cur-
increase Na+ excretion and decrease K+ ex-
rent via voltage-gated channels that leads to
cretion so that K+ concentrations will be de-
the plateau. The result is a slow conduction ve-
creased in the urine and Na+ concentrations
locity that prolongs the transmission from the
will be increased. Treatment involves cystidine
atria to the ventricles.
or uridine to bypass this step in pyrimidine
synthesis and also to negatively downregulate Answer A is incorrect. Ion channels in the
orotic acid production. myocardium are voltage gated.
Answer E is incorrect. Spironolactone de- Answer B is incorrect. During phase 2,
Cardiovascular
creases K+ excretion but increases Na+ excre- voltage-gated potassium cells open to allow po-
tion; therefore, Na+ concentrations will be el- tassium efflux.
evated in the urine. Answer C is incorrect. Ion channels in the
myocardium are voltage gated.
5. The correct answer is A. This patient has te-
tralogy of Fallot, which is defined by the com- Answer D is incorrect. Voltage-gated sodium
bined symptoms of ventricular septal defect channels are responsible for the upstroke in
(VSD), overriding aorta, pulmonary stenosis, ventricular cells (phase 0).
and right ventricular hypertrophy. The cause Answer E is incorrect. During phase 2,
of these abnormalities, however, is an antero- voltage-gated potassium cells open to allow po-
superior displacement of the infundibular tassium efflux.
septum during heart development in utero.
Patients with tetralogy of Fallot learn to squat Answer F is incorrect. Closing voltage-gated
during cyanotic spells, which causes compres- sodium channels would hyperpolarize the cell.
sion of the femoral arteries, thereby decreases Voltage-gated sodium channels are responsible
their right-to-left shunt. for the upstroke in ventricular cells (phase 0).
These open in response to depolarization to
Answer B is incorrect. An overriding aorta is the -55mV threshold value, allowing sodium
one of the four manifestations of tetralogy of to rapidly flow down its concentration gradient
Fallot, not the cause. into the cell. These channels are then inacti-
176 Section II: Organ Systems • Answers
vated and cannot be opened again until the 8. The correct answer is A. Epinephrine is a
cell is repolarized. nonselective agonist of a- and b-adrenergic
receptors. Administering a large dose of epi-
7. The correct answer is E. Troponin is a protein nephrine causes an increase in blood pressure
found along the sarcomeres that assist in mus- via an increased heart rate and in contractil-
cle contraction. With muscle injury, troponin ity through stimulation of b1 receptors; and
is leaked into the serum. Different fractions net a increase in systemic vascular resistance
show different specificities for different tissues. through a1-mediated vasoconstriction (the
In cardiac tissue, troponin I has been shown to b2-mediated vasodilation is negligible com-
be more specific and equally if not more sensi- pared to the a1 effects). Adding phentolamine,
tive than cardiac enzymes, CK-MB in particu- a selective a1 antagonist, blocks the a effects
lar. of epinephrine. Therefore re-administration
High-Yield Systems
Answer A is incorrect. Alanine aminotransfer- leaves only the b1-receptor actions (increased
ase (ALT) is found mostly in the liver and is contractility and heart rate) and the b2-
therefore a specific marker of damage to hep mediated increase in vasodilation, causing a
atocytes. Elevations of ALT are not associated net decrease in blood pressure.
with myocardial infarction (MI). Answer B is incorrect. Isoproterenol is an ago-
Answer B is incorrect. Aspartate aminotrans- nist of b- and a-adrenergic receptors, although
ferase (AST) is an enzyme found in the heart, its primary action is at the b receptor. Hence,
liver, and skeletal muscle, and an elevation in adding isoproterenol actually would cause a
it is nonspecific for damage to those tissues. decrease in pressure through b2-mediated vaso-
Although commonly elevated in the setting of dilation. Clonidine is an a agonist, and would
liver disease, AST levels peak around day two lead to decreased sympathetic outflow and pos-
post-MI. sibly cause an additional decrease in pressure.
Adding isoproterenol after clonidine admin-
Answer C is incorrect. Creatinine kinase (CK) istration would lead to a further decrease in
is located on the inner mitochondrial mem- blood pressure.
brane, on myofibrils, and in the muscle cyto-
Answer C is incorrect. Norepinephrine is an
Cardiovascular
in pressure after repeat phenylephrine admin- Answer A is incorrect. The anterior papillary
istration. muscle is supplied by both the left anterior de-
scending artery and the left circumflex artery.
9. The correct answer is E. Nifedipine is a dihy- Because of its dual blood supply, the anterior
dropyridine class calcium channel blocker that papillary muscle is less likely to rupture after
could be used in the long-term control of hy- a MI. Furthermore, the patient’s initial MI in-
pertension. However, in the case of a hyperten- volved the posterior aspect of the myocardium.
sive emergency, nifedipine used sublingually
Answer B is incorrect. The anterior papillary
can cause dangerous fluctuations in blood
muscle is supplied by both the left anterior de-
pressure that are difficult to control and can
scending artery and the left circumflex artery.
lead to more harm than good.
Because of its dual blood supply, the anterior
Answer A is incorrect. Diltiazem is a benzo- papillary muscle is less likely to rupture after
High-Yield Systems
thiazepine class calcium channel blocker that a MI. Furthermore, the patient’s initial MI in-
reduces myocardial demand and also causes volved the posterior aspect of the myocardium.
vasodilation. It is not contraindicated in this
Answer C is incorrect. The left (or obtuse)
patient.
marginal artery, which is a branch of the left
Answer B is incorrect. Labetalol is a com- circumflex artery, follows the left border of the
bined a/b-blocker that has effects on both re- heart to supply the left ventricle.
ceptors. It can be used in a hypertensive situa-
Answer E is incorrect. The right marginal ar-
tion as an emergent option for treatment. It is
tery follows the inferior border of the heart to
not contraindicated in this patient.
supply the right ventricle.
Answer C is incorrect. Metoprolol is a
b-blocker used to treat angina by reducing 11. The correct answer is C. The key finding is
heart rate and contractility. It also reduces the hypertrophy with asymmetric septal enlarge-
metabolic demand of the myocardium. It is ment without free ventricular wall enlarge-
often used to control hypertension, but is not ment. This is seen in patients with hyper
contraindicated in this patient. trophic obstructive cardiomyopathy (HOCM).
Cardiovascular
Mutation in a sarcomere gene such as myosin-
Answer D is incorrect. Captopril is an ACE
binding protein C is one of the most common
inhibitor used in the control of chronic hyper-
genetic causes of HOCM. This type of cardio-
tension. It is especially useful for patients who
myopathy is inherited in an autosomal domi-
have signs of renal disease and can slow the
nant fashion. The anatomic distortion can lead
progression of damage to the kidneys. It is not
to a dynamic ventricular outflow obstruction
contraindicated in this patient.
during systolic ejection, which leads to a sys-
10. The correct answer is D. This patient most tolic murmur, dyspnea, lightheadedness, syn-
likely suffered from acute mitral regurgitation cope, and in many cases sudden death. On
secondary to rupture of the posterior papillary histologic examination, HOCM would show
muscle. The anterior and posterior papillary disoriented, tangled, hypertrophied myocardial
muscles anchor the chordae tendineae, which fibers.
prevent the cusps of the mitral valve from be- Answer A is incorrect. Infection with an En-
ing forced into the left atrium. An occlusion terovirus, namely coxsackie B virus, initially
of the posterior descending artery can lead to causes a subacute myocarditis but can eventu-
an infarction of the posterior papillary muscle ally lead to dilated cardiomyopathy (DCM).
and subsequent rupture of the muscle several Pathologically, DCM produces a grossly en-
days later. Patients will present with a sudden larged heart with dilation of all four chambers
onset of pulmonary edema and frequently car- and normal ventricular wall thickness. Other
diogenic shock. etiologies for DCM include alcohol abuse, wet
beriberi, cocaine use, Chagas disease, doxoru-
178 Section II: Organ Systems • Answers
bicin toxicity, hemochromatosis, and peripar- Answer D is incorrect. Proteins C and S act as
tum cardiomyopathy. negative regulators of the coagulation cascade.
Therefore a deficiency, rather than overpro-
Answer B is incorrect. Immune-mediated de-
duction, will lead to a hypercoagulable state.
struction of cardiomyocytes is the cause of car-
diac damage in rheumatic fever. This disease Answer E is incorrect. Von Willebrand factor
is a consequence of pharyngeal infection with allows platelets to adhere to a defect where col-
group A b-hemolytic streptococci. Early deaths lagen is exposed and binds inactive factor VIII
from rheumatic heart disease are due to myo- in circulation. A deficiency (von Willebrand
carditis, whereas late sequelae include damage disease) leads to bleeding complications such
to the heart valves. Histologically, this disease as epistaxis, menorrhagia, and gastrointestinal
is associated with Aschoff bodies, which are (GI) bleeds.
granulomas with giant cells. The heart muscle
High-Yield Systems
itself, however, would appear grossly normal. 13. The correct answer is E. This girl has
Takayasu arteritis, a large-vessel vasculitis usu-
Answer D is incorrect. Deposits of protein
ally found in women <40 years of age of Asian
such as light chains, heavy chains, or trans-
descent. It is characterized by a thickening of
thyretin are associated with amyloidosis. Amy-
the aortic arch and/or the proximal great ves-
loidosis produces a restrictive cardiomyopathy,
sels. The most prominent feature is weak
in which the ventricular wall and chamber
pulses in the upper extremities. Complications
size grossly appear normal, which is not con-
of untreated late-stage Takayasu include aor-
sistent with the reduced chamber size seen in
tic aneurysms and typically involve the aortic
this specimen. In addition, amyloidosis is gen-
arch. This leads to narrowing, or possible oblit-
erally a disease of the elderly, and is thus not as
eration, of the major arteries associated with
likely in this 17-year-old patient.
the aortic arch. Treatment involves high doses
Answer E is incorrect. Chagas disease is of oral prednisone that are tapered over many
caused by a trypanosome that is primarily months as the clinician and patient agree to
endemic to South America. Eighty percent minimize the adverse effects of corticosteroids.
of those infected will develop myocarditis,
Answer A is incorrect. Abdominal pain and
Cardiovascular
ticosteroids. Giant-cell arteritis may present as diac cycle; in other words, it slows the diastolic
sudden monocular blindness, and emergent depolarization of pacemaker cells.
corticosteroids are needed to save that eye.
Answer E is incorrect. Bepridil, not verapamil,
Answer C is incorrect. Pneumonitis is a clas- is a calcium channel blocker and class IV an-
sic symptom of Wegener granulomatosis, a tiarrhythmic known to be associated with tor-
necrotizing small-vessel vasculitis primarily af- sades de pointes. Verapamil and diltiazem are
fecting the kidneys and the lungs. The classic two calcium channel blockers used in the pre-
clinical vignette for Wegener will describe a vention of nodal arrhythmias (eg, supraventric-
patient who presents with hemoptysis and he- ular tachycardia).
maturia and who has classical antineutrophil
cytoplasmic antibodies on serologic evaluation. 15. The correct answer is E. This patient presents
with shortness of breath and dyspnea on exer-
Answer D is incorrect. Strawberry tongue is
High-Yield Systems
tion. The bilateral crackles detected on physi-
seen in children with Kawasaki disease, an ar-
cal examination indicate pulmonary edema.
teritis that often involves the coronary arteries,
This spectrum of signs and symptoms are
but can affect vessels of any size. In children,
suggestive of left ventricular dysfunction lead-
the symptoms of Kawasaki’s can relent with-
ing to CHF. CHF itself has many causes, but
out treatment and the major complications
the lack of other clinical findings (no rubs or
are coronary artery aneurysms and MIs. The
murmurs) indicates either hypertensive heart
diagnostic criteria for this disease are five days
disease or cardiomyopathy as likely etiologies.
of fever plus four of five of the following: (1)
Cardiomyopathy can be divided into dilated,
erythema of the lips or oral cavity or cracking
hypertrophic, or restrictive. Alcoholism and
of the lips; (2) rash on the trunk; (3) swelling
coxsackie virus B have been strongly associated
or erythema of the hands or feet; (4) red eyes
with dilated cardiomyopathy.
(conjunctival injection); and (5) a swollen
lymph node in the neck >15 mm. Answer A is incorrect. Aortic strictures are
typically associated with the congenital de-
14. The correct answer is D. The ECG demon- fect known as coarctation of the aorta. Twice
strates a pattern characteristic of torsades de as common in men as in women, coarctation
Cardiovascular
pointes. Quinidine, a class IA antiarrhythmic is fairly common in patients with Turner syn-
agent, is used to treat supraventricular arrhyth- drome. While pumping against a permanently
mias by slowing conduction. A simultaneous elevated afterload may cause left ventricular
increase of the QT interval caused by quini- hypertrophy and eventually failure, this condi-
dine risks torsades de pointes. tion would result in a slightly different clinical
picture. The patient would have higher blood
Answer A is incorrect. Adenosine is used
pressure in the upper extremities than in the
both to diagnose and to treat supraventricular
lower extremities, causing additional symptoms
tachyarrhythmias. However, it is not associated
of claudication and coldness in the feet. Also,
with torsades des pointes.
coarctations often produce a detectable sys-
Answer B is incorrect. Although the class III tolic murmur.
antiarrhythmics tend to be associated with tor-
Answer B is incorrect. Fibrosis of the endocar-
sades des pointes, bretylium is an exception to
dium is associated with the effects of a carci-
this rule and has no association with this con-
noid tumor on the heart. Signs that are usually
dition.
detected on auscultation are pulmonic stenosis
Answer C is incorrect. Propranolol is a class II or tricuspid regurgitation. Classic symptoms of
antiarrhythmic, but it is not associated with tor- carcinoid syndrome include flushing, diarrhea,
sades de pointes. b Blockers such as proprano- and bronchospasm.
lol are used to suppress abnormal pacemakers
Answer C is incorrect. Fibrotic debris within
by decreasing the slope of phase 4 of the car-
the pericardial membrane occurs in constric-
180 Section II: Organ Systems • Answers
tive pericarditis. A pericardial friction rub Answer E is incorrect. Vasodilation will re-
would be detectable on auscultation. duce blood pressure, and pregnant women do
have a constant amount of vasodilation that
Answer D is incorrect. Ischemia of the myo-
is greater than that in nonpregnant women.
cardium is usually the result of coronary artery
Blood pressure decreases by about 10 mm Hg
disease and typically presents with complaints
during pregnancy, particularly during the first
of chest pain. An active coronary artery disease
and second trimesters. However, this vasodila-
process has been ruled out by angiography.
tion should remain relatively constant when
16. The correct answer is C. Inferior vena cava transferring from an upright to a supine posi-
(IVC) compression is common in women dur- tion, and would not account for the sudden hy-
ing the third trimester of pregnancy. The large potension seen in this case.
uterus compresses the IVC, decreasing venous
High-Yield Systems
greater than the increase in RBC mass, result- various tissues, most notably the heart.
ing in a physiologic anemia. However, this
Answer A is incorrect. Endothelial dysfunc-
generally does not cause adverse symptoms for
tion is one of the primary steps in the patho-
the mother, and would not cause hypotension
genesis of atherosclerosis. Endothelial dysfunc-
in a supine position.
tion can be caused by numerous factors, such
Answer B is incorrect. Cardiogenic shock can as the toxins found in cigarette smoke, and is
cause hypotension by decreasing the stroke not necessarily caused by oxidized LDL cho-
volume and cardiac output, but it would not lesterol. However, when looking specifically at
occur only in the supine position. the relationship between hyperlipidemia and
Answer D is incorrect. When fluid leaves the atherosclerosis, LDL oxidation generally oc-
intravascular space and enters the interstitial curs before endothelial dysfunction.
space, it is referred to as third spacing. In preg- Answer B is incorrect. Foam cells are formed
nancy, there is a physiologic amount of third from macrophages that phagocytosed oxi-
spacing, which causes dependent edema in the dized LDL cholesterol. In fact, they only up-
hands and feet. Some women may even expe- take LDL cholesterol once it has undergone
rience pulmonary edema, which can be dan- oxidation, which then causes changes in the
gerous. Third spacing does cause hypotension LDL cholesterol that signal activation of mac-
if the intravascular volume is not replaced, but rophage “scavenger” receptors (CD36). Foam
it would not cause isolated hypotension in the cells play an important role in the pathogen-
supine position. esis of atherosclerosis, but they are not the first
Chapter 8: Cardiovascular • Answers 181
High-Yield Systems
18. The correct answer is A. In tricuspid regurgi- would increase, not decrease, with mitral ste-
tation, blood flows backward into the atria dur- nosis.
ing ventricular systole. This would affect the Answer B is incorrect. Mitral stenosis would
C and X waves, replacing them with a large not result in an overall decrease in left ventric-
positive deflection. This positive deflection ef- ular pressure; pressure will only be decreased
fectively joins the C wave and the V wave, cre- relative to the left atrium.
ating the “CV wave.” The C wave is thought to
Answer D is incorrect. Left ventricular pres-
be due to pressure on the tricuspid valve dur-
sure would increase with aortic stenosis. Rheu-
ing ventricular systole. If the valve allows back-
matic heart disease most typically presents as
flow during ventricular systole, the increased
mitral stenosis, although aortic stenosis is the
ventricular pressures would be transmitted
next most common presentation.
back into the right atrium and the jugular
vein. The downward movement of the ventri- Answer E is incorrect. Right atrial pressure
cle causes the x descent during ventricular sys- would increase in response to pulmonic valve
tole. This would also be replaced by a positive stenosis or tricuspid valve stenosis. The tricus-
deflection from blood regurgitating into the pid valve is sometimes affected in rheumatic
Cardiovascular
atria during ventricular systole. heart disease and would also cause a diastolic
murmur. However, this is far less common
Answer B is incorrect. These points are not
than mitral stenosis in the setting of rheumatic
the most likely to be affected in tricuspid re-
heart disease. In fact, the pulmonic valve is the
gurgitation.
least likely to be affected in rheumatic heart
Answer C is incorrect. These points are not disease.
the most likely to be affected in tricuspid re-
gurgitation. 20. The correct answer is B. The inferior mesen-
teric artery (IMA) originates from the aorta in-
Answer D is incorrect. These points are not
ferior to the renal arteries and superior to the
the most likely to be affected in tricuspid re-
bifurcation of the aorta into the common iliac
gurgitation. The V wave is increased pressure
arteries. The IMA supplies blood to the distal
because of right atrial filling against a closed
one-third of the transverse colon, descending
tricuspid valve. With tricuspid regurgitation,
and sigmoid colons, and the upper portion of
there is little effect on the V wave itself, but
the rectum. The IMA may sometimes be sacri-
rather it becomes the end point for the new
ficed during an infrarenal aortic aneurysm re-
CV-wave change.
pair rather than being reattached to a healthy
19. The correct answer is C. The patient’s history segment of aorta. Usually there is enough col-
of a prior illness with features of fever, pleu- lateral flow to the hindgut from the superior
ritic chest pain, joint pain, and rash (probably mesenteric artery and the hypogastric arteries
that the loss of the IMA does not result in co-
182 Section II: Organ Systems • Answers
lonic ischemia. However, ischemia of the sig- Answer B is incorrect. Congenital medial
moid colon occurs in 1%-7% of repairs, and weakness is actually associated with the devel-
should be considered if bloody diarrhea or an opment of berry aneurysms, which typically
increased WBC count occurs postoperatively. occur along the circle of Willis. They are the
most frequent cause of subarachnoid hemor-
Answer A is incorrect. The ascending colon
rhage and are also associated with adult poly-
receives its blood supply from the superior
cystic kidney disease.
mesenteric artery. This vessel is located supe-
rior to the renal arteries and thus would not Answer C is incorrect. Cystic medial necrosis
be disrupted during resection of the infrarenal (cystic degeneration of the tunica media of the
aorta. aorta) is the most frequent pre-existing histo-
logic lesion in aortic dissection. It is associated
Answer C is incorrect. The small intestine, as
with dilation of the ascending aorta, particu-
High-Yield Systems
High-Yield Systems
begins to undergo coagulative necrosis and
by reducing calcium current. As a result, the
releases enzymes such as troponin I and CK-
conduction through the AV node is markedly
MB from the dying cells. Coagulative necrosis
reduced. In addition to this, adenosine’s ex-
is marked in the early stages by preservation of
tremely short duration of action (15 seconds)
general tissue architecture, with myocytes be-
limits the occurrence of its toxicities (ie, hypo-
coming increasingly eosinophilic. Contraction
tension, flushing, chest pain, and dyspnea).
bands will also be seen, causing myocytes to
Answer B is incorrect. Bretylium, a potassium take on a wavy appearance.
channel blocker (class III), is used when other
Answer B is incorrect. The presence of granu-
antiarrhythmics fail.
lation tissue indicates that remodeling of dam-
Answer C is incorrect. Encainide is used aged tissue is occurring. It is characterized by
when ventricular tachycardia progresses to the presence of fibroblasts and vascular prolif-
ventricular fibrillation; it is also used in intrac- eration. This would first be seen, along with a
table supraventricular tachycardia. mixed inflammatory picture, beginning 5-10
Answer D is incorrect. Lidocaine, a class Ib days after a MI.
antiarrhythmic, is used in the treatment of Answer C is incorrect. Macrophages are seen
Cardiovascular
acute ventricular arrhythmias such as post-MI in the development of chronic inflammation.
arrhythmias. Five to ten days after an MI, macrophages
Answer E is incorrect. Sotalol, which is both come to the scene to aid in cleaning up areas
a b-adrenergic-receptor blocker (class II) and a of dead tissue. It is important to remember
potassium channel blocker (class III), is used that it usually takes time for macrophages and
when other antiarrhythmics fail. other signs of chronic inflammation to appear
in any setting.
24. The correct answer is E. Gemfibrozil is a fi- Answer D is incorrect. Neutrophils are the
brate that stimulates lipoprotein lipase by ac- hallmark of an acute inflammatory process. A
tivation of the PPARa protein. In so doing, it massive influx of neutrophils, along with ex-
has beneficial effects on the serum lipid pro- tensive coagulative necrosis, begins at 12-24
file. Adverse effects of fibrates include eleva- hours and continues until about four days af-
tion of liver enzymes and myositis. ter an MI. Although neutrophils are generally
Answer A is incorrect. Gemfibrozil speeds the considered the first responders in any inflam-
conversion of VLDL to LDL cholesterol. matory process, they appear after the start of
coagulative necrosis in the evolution of an MI.
Answer B is incorrect. Gemfibrozil has no ef-
fect on triglyceride receptors. Answer E is incorrect. In the first 2-4 hours
following an MI, no change can be de-
Answer C is incorrect. Gemfibrozil promotes tected using light microscopy. However, by
PPAR-a activation.
184 Section II: Organ Systems • Answers
eight hours changes associated with coagula- are stiff. In addition, in some cases a third
tive necrosis would certainly be visible. heart sound (S3) may be heard shortly after
point A at the beginning of diastole. S3 is due
26. The correct answer is B. Point A corresponds to the vibration of the distended ventricular
to the opening of the mitral valve at the be- wall during rapid filling and is usually soft and
ginning of diastole, and the line from A to C low in frequency. While the presence of an S3
shows the increase in ventricular volume dur- is normal in children, in adults it usually sug-
ing diastole. Point C marks the beginning of gests volume overload, such as occurs in CHF.
systole as left ventricular pressure becomes
Answer C is incorrect. Point C corresponds to
greater than left atrial pressure, causing the
S1, which is heard normally when the mitral
mitral valve to close. This closure (in conjunc-
and tricuspid valves close at the end of dias-
tion with the closure of the tricuspid valve)
tole. The S4 sound would be heard just before
High-Yield Systems
end of ventricular systole. When the left ven- The closure of the aortic valve (in conjunction
tricular pressure becomes less than the pres- with the closure of the pulmonic valve) can
sure in the left atrium, the mitral valve opens, be heard on auscultation as the second heart
thus beginning a new loop of the cardiac cycle sound (S2).
(diastole plus systole). Point B corresponds
to the point near the end of diastole when S4 27. The correct answer is B. The patient is expe-
may be heard. An S4, commonly called the riencing pericarditis due to uremia second-
“atrial kick,” is not normally present in adults. ary to chronic kidney disease in the setting of
Its presence suggests a decrease in ventricular long-standing diabetes mellitus (DM). Pericar-
compliance, such as occurs in ventricular hy- ditis presents with pleuritic, positional chest
pertrophy resulting from chronic hypertension. pain that is often relieved by sitting forward
S4 is thought to result from vibration of a stiff, and with a pericardial friction rub on physical
noncompliant ventricular wall as blood is rap- examination. Diffuse ST segment elevations
idly ejected into the ventricle from the atrium. may be found on ECG, while an echocardio-
gram may be normal unless an effusion is also
Answer A is incorrect. Point A represents the present. Pericarditis has multiple etiologies,
opening of the mitral valve at the beginning of including viral (coxsackie virus, echovirus, ad-
diastole, not an S4 heart sound. Normally no enovirus, and HIV), bacterial (tuberculosis or
sound is heard when the mitral valve opens. Streptococcus pneumoniae or Staphylococcus
However, in cases of mitral stenosis, an open- aureus in the setting of endocarditis, pneumo-
ing click may be audible if the valve leaflets nia, or post-cardiac surgery), neoplastic, auto-
Chapter 8: Cardiovascular • Answers 185
immune, uremic, cardiovascular, or idiopathic. There are four general classes of antiarrhyth-
Treatment of pericarditis secondary to uremia mics: class I blocks sodium channels, class II
is dialysis. blocks b-adrenergic receptors, class III blocks
potassium channels, and class IV blocks cal-
Answer A is incorrect. Cardiac catheteriza-
cium channels. Epinephrine, amiodarone
tion is indicated in patients who are experi-
(class IA and class III properties), or lidocaine
encing acute coronary syndrome. While this
(class IB) are the agents indicated after a round
patient is certainly at risk for ischemic heart
of unsuccessful defibrillation. Of these agents,
disease given her age and history of diabetes,
only amiodarone may produce the adverse ef-
her symptoms and ECG findings of diffuse ST
fect of pulmonary fibrosis. Amiodarone is also
segment elevations are more indicative of peri-
associated with hypotension, thyroid dysfunc-
carditis. In contrast, acute myocardial ischemia
tion (both hypo- and hyperthyroidism), hepa-
is more likely to present with chest pain that is
High-Yield Systems
totoxicity, ocular changes, and other arrhyth-
not relieved by changes in position and ECG
mias (namely, bradyarrhythmias and torsades
findings that show ST segment elevations in
de pointes). In patients taking amiodarone,
contiguous leads only.
remember to check pulmonary function tests,
Answer C is incorrect. NSAIDs are the ap- liver function tests, and thyroid function tests.
propriate treatment for viral or idiopathic peri-
Answer A is incorrect. There is no increased
carditis. However, this patient is experienc-
risk of bleeding associated with any of the an-
ing pericarditis due to uremia secondary to
tiarrhythmic drugs. Warfarin and heparin are
chronic kidney disease in the setting of long-
commonly used drugs that can cause bleeding.
standing DM, and therefore the most effective
However, they would not be used during ven-
treatment for her is dialysis.
tricular fibrillation.
Answer D is incorrect. Pericardiocentesis
Answer B is incorrect. Class IC agents (ie, fle-
would be indicated if this patient had evidence
cainide and encainide) are contraindicated in
of a pericardial effusion, such as distant heart
the post-MI population, due to their associa-
sounds on physical examination, electrical al-
tion with increased post-MI mortality. These
ternans on ECG, cardiomegaly on x-ray of the
drugs work at phase 3 of the action potential
Cardiovascular
chest, or fluid in the pericardial space on echo-
and have no effect on action potential dura-
cardiogram. However, this patient does not
tion. They are most often used in cases of ven-
have any of these findings, and therefore peri-
tricular tachycardia that progress to ventricu-
cardiocentesis is not indicated.
lar fibrillation, or intractable supraventricular
Answer E is incorrect. Changing the patient’s tachycardia.
antibiotic regimen is indicated if she is sus-
Answer C is incorrect. Procainamide, a type
pected of having an allergic reaction (such as
1A antiarrhythmic drug, is a common cause of
the development of a rash or anaphylaxis) to
drug-induced lupus, but it is not used to treat
a particular medication. However, in this case
a patient in ventricular fibrillation. It acts by
her symptoms are not consistent with an aller-
increasing action potential duration and in-
gic reaction, and therefore her antibiotic regi-
creasing QT interval. It is most often used for
men does not need to be changed.
treatment of re-entrant and ectopic supraven-
28. The correct answer is D. Ventricular fibrilla- tricular and ventricular tachycardia. Other
tion is an irregular ventricular rhythm without drugs in this class include quinidine (which
any distinct QRS complexes, ST segments, or has the adverse effect of cinchonism), amioda-
T waves. This is an important cause of sud- rone (which is also a class III antiarrhythmic),
den cardiac death, as well as mortality within and disopyramide.
the first 24 hours of an acute MI. If defibrilla- Answer E is incorrect. Digoxin toxicity can
tion fails to convert to sinus rhythm, the next lead to disturbances in color vision, including
treatment choice is the use of antiarrhythmics. yellow-green vision, as well as nausea, vomit-
186 Section II: Organ Systems • Answers
ing, diarrhea, and arrhythmias. Digoxin di- gram-negative bacilli (Haemophilus aphrophi-
rectly inhibits Na+/K+-ATPase, leading to in- lus, Actinobacillus actinomycetemcomitans,
direct inhibition of the Na+/Ca2+ exchanger. Cardiobacterium hominis, Eikenella corrodens,
Therefore there is an increase in intracellular and Kingella kingae) that cause 5%-10% of
calcium, leading to positive inotropy. This cases of bacterial endocarditis that are not re-
drug is most often used for chronic heart fail- lated to IV drug use. These organisms are slow
ure and control of atrial fibrillation, not for growing and difficult to culture from blood
ventricular fibrillation. samples, making diagnosis more complex.
Answer D is incorrect. Streptococcus bovis also
29. The correct answer is C. This is a classic case
causes subacute bacterial endocarditis, which
of acute bacterial endocarditis (ABE). Endo-
presents with low-grade fever and insidious on-
carditis often is characterized by constitutional
set. It normally inhabits the lower GI tract, and
High-Yield Systems
septic emboli to the lungs, leading to bilateral dures during which normal flora of the oro-
infiltrates. This patient is manifesting signs of pharynx can enter the bloodstream.
bilateral infiltrates with hypoxia, decreased
breath sounds, and dullness to percussion. It 30. The correct answer is B. Angiotensin II recep-
is important to note that many of the classic tor blockers such as losartan have similar blood
signs of endocarditis, such as Janeway lesions, pressure-lowering properties as ACE inhibi-
Osler’s nodes, and Roth’s spots, are seen mostly tors, but have fewer adverse effects. Adverse ef-
as a complication of left-sided endocarditis, in fects of angiotensin II receptors include hyper-
which septic emboli leave the heart and enter kalemia and fetal renal toxicity.
the systemic circulation. Answer A is incorrect. Blockade of b1 recep-
Answer A is incorrect. Enterococcus faecalis tors is the mechanism of action of acebutolol,
also causes subacute endocarditis. The classic betaxolol, esmolol, atenolol, and metoprolol.
picture is a slow onset of constitutional symp- These drugs are used to treat hypertension,
toms with low-grade fever. Enterococcus infec- in addition to angina, supraventricular tachy-
tion is not seen as frequently as Streptococcus cardia, CHF, and glaucoma. These drugs also
viridans, but it is known to colonize damaged decrease post-MI mortality. Common toxicities
heart valves, especially in patients with a his- include impotence, exacerbation of asthma,
tory of rheumatic fever. sedation, bradycardia, and atrioventricular
block.
Answer B is incorrect. Haemophilus aphrophi-
lus is part of the HACEK group of fastidious
Chapter 8: Cardiovascular • Answers 187
Answer C is incorrect. ACE inhibitors, such tolic blood pressure drops by >10 mm Hg on
as captopril and enalapril, are antihypertensive inspiration. ECG is often low voltage and may
drugs commonly prescribed to diabetics due to reveal electrical alternans. Because patients in
their possible renal-sparing properties. How- cardiac tamponade are in a low-output state,
ever, these drugs have common adverse effects they are preload dependent and require imme-
such as hyperkalemia, cough, angioedema, diate volume resuscitation to maintain cardiac
taste changes, hypotension, and rash. In addi- output. Positive inotropes such as dobutamine
tion, ACE inhibitors are teratogenic, causing and pericardiocentesis would also be indicated
fetal renal problems. This patient was taking in this patient after he has begun receiving IV
an ACE inhibitor before. The adverse effects hydration.
of cough and angioedema prompted her physi-
Answer A is incorrect. Diltiazem is a calcium
cian to switch her to an angiotensin II receptor
channel blocker that has a negative inotropic
High-Yield Systems
blocker.
effect on the heart. In the setting of cardiac
Answer D is incorrect. Nifedipine, verapamil, tamponade, a negative inotrope like diltiazem
and diltiazem are drugs that act through in- is contraindicated because it would decrease
hibition of calcium channels in cardiac and his already low cardiac output and therefore
smooth muscle. They are commonly used worsen his hypotension and shock.
in the treatment of hypertension, angina,
Answer B is incorrect. Metoprolol is a selec-
Prinzmetal’s angina, and Raynaud syndrome.
tive b1-blocker that has negative inotropic ef-
Verapamil and diltiazem are also used as anti-
fects on the heart. In the setting of cardiac
arrhythmics.
tamponade, a negative inotrope like meto-
Answer E is incorrect. This is the mechanism prolol is contraindicated because it would
of action of thiazide diuretics such as hydro- decrease his already low cardiac output and
chlorothiazide, which are commonly used an- therefore worsen his hypotension and shock.
tihypertensive agents. Adverse effects include
Answer C is incorrect. Because patients in
hypokalemic metabolic alkalosis, hyponatre-
cardiac tamponade are in a low-output state
mia, hyperglycemia, hyperlipidemia, hyperuri-
due to the compression of the heart by the sur-
cemia, hypercalcemia, and allergic reactions.
Cardiovascular
rounding fluid within the pericardial sac, their
cardiac output is preload dependent. Any in-
31. The correct answer is D. This patient is suffer-
tervention that decreases his preload would be
ing from cardiogenic shock due to pericardial
contraindicated in this setting because it would
tamponade secondary to his small cell lung
lead to decreased cardiac output and worsen-
cancer. Cardiac tamponade can occur sec-
ing hypotension and shock; therefore, diuresis
ondary to trauma, hypothyroidism, myocardial
is not indicated in this patient.
rupture, or as a complication of pericarditis
(especially in the setting of malignancy or ure- Answer E is incorrect. In the setting of cardiac
mia). Specifically, cardiac tamponade results tamponade, surgery is indicated only if fluid
when the pericardial space fills with enough has reaccumulated after catheter drainage, the
fluid to cause increased intrapericardial pres- effusion is loculated, there is a special need for
sure, compression of the heart throughout its biopsy material, or the patient has a coagulopa-
cycle, and subsequent decreased diastolic fill- thy. Moreover, general anesthesia is usually re-
ing of the heart. As a result of the decreased quired, and may be unsafe if needle drainage
preload, stroke volume falls and cardiogenic is not performed first to reduce the severity of
shock (in the absence of pulmonary edema) the tamponade. Therefore, surgery is not the
results. Classic physical examination findings most appropriate next step in the management
in cardiac tamponade include Beck’s triad of of this patient.
distant heart sounds, increased jugular venous
pressure, and hypotension. Pulsus paradoxus 32. The correct answer is A. This patient presents
may also be seen, which occurs when the sys- with symptoms consistent with Lyme disease.
188 Section II: Organ Systems • Answers
He had a characteristic expanding rash (ery- Answer A is incorrect. VSD causes a harsh
thema migrans) and resolving flu-like symp- holosystolic murmur heard best over the tricus-
toms. Lyme disease can often lead to cardiac pid area. VSD is a relatively common congeni-
symptoms such as those described, as well tal cardiac anomaly; however, this murmur
as heart block that can require cardiac pac- would not be heard in heart failure, which is
ing. Lyme disease is carried by the Ixodes tick. the most likely disease process occurring in
I scapularis is also the vector of disease for ba- this case.
besiosis, a malaria-like parasitic disease com-
Answer B is incorrect. In the absence of dis-
mon in the northeastern corner of the United
ease, the sounds made by the closing of the
States.
aortic and pulmonic valves (S2) occur simul-
Answer B is incorrect. Epidemic typhus is taneously during expiration, but are split dur-
known as “louse-borne typhus.” The vector for ing inspiration as the decrease in intrathoracic
High-Yield Systems
transmission is Pediculus corporis. Epidemic ty- pressure causes a delay in the closing of the
phus is unusual because the vector for disease pulmonic valve. Paradoxical splitting occurs
feeds only on humans and not other animals. in cases of aortic stenosis or left bundle branch
The bacterium responsible is Rickettsia prowa- block, when the closing of the aortic valve is
zekii. delayed and thus the pulmonic valve closes
before the aortic valve on expiration, but the
Answer C is incorrect. Malaria is a protozoan
delayed closure of the pulmonic valve on in-
parasitic disease responsible for one-three mil-
spiration causes the sounds to be simultaneous
lion deaths per year worldwide. Its vector of
on inspiration. This patient does not have any
transmission (and target for disease control) is
signs or symptoms of aortic stenosis or a left
the female Anopheles mosquito.
bundle branch block, but she does have signs
Answer D is incorrect. Plague is an infectious of volume overload due to CHF. Thus she
disease caused by the bacterium Yersinia pes- would likely have an abnormal S3, not a para-
tis. It is mainly transmitted by fleas that live on doxically split S2.
infected rodents such as the oriental rat flea,
Answer C is incorrect. A pulmonary flow mur-
Xenopsylla cheopis.
mur is a systolic murmur heard best over the
Cardiovascular
Answer E is incorrect. Rocky Mountain spot- pulmonic area, associated with increased flow
ted fever is caused by Rickettsia rickettsii, a spe- across the pulmonary valve. This occurs in
cies of bacteria spread to humans by the ticks conditions such as an ASD, in which blood
of the Dermacentor family such as D variabilis. from the left heart flows to the right heart,
thus increasing the volume of blood that flows
33. The correct answer is D. This woman has through the pulmonic valve. ASD can also
CHF, which is causing pulmonary and sys- cause a diastolic rumble due to increased flow
temic edema due to elevated venous pressures over the tricuspid valve. However, an ASD
(“backward failure”) and fatigue due to an in- murmur would not be heard in this clear case
sufficient cardiac output to meet the metabolic of heart failure.
demands of the body (“forward failure”). In an
attempt to compensate for the decreased car- Answer E is incorrect. The fourth heart sound
diac output, the heart operates at higher end- (S4) occurs in late diastole and coincides with
diastolic and end-systolic volumes, which often atrial contraction in cases in which the atrium
produces a third heart sound (S3), most likely contracts against a stiffened ventricle. An S4 is
due to the increased tension of the chordae not present in normal children or adults, and
tendinae during the rapid filling phase of early suggests a decrease in ventricular compliance,
ventricular diastole. While the presence of an as is seen in the ventricular hypertrophy that
S3 is normal in children, in adults it is often a develops in chronic hypertension. This pa-
sign of volume overload, as in CHF. tient has signs of volume overload and would
be more likely to present with an S3 due to
Chapter 8: Cardiovascular • Answers 189
High-Yield Systems
normal splitting of S2 during inspiration. S2 35. The correct answer is D. This infant has a pat-
may be audibly split during expiration as well, ent ductus arteriosus (PDA). The sixth aortic
as the pulmonic valve closes after the aortic arch gives rise to the proximal pulmonary ar-
valve, regardless of respiratory cycle. teries and, on the left side, to the ductus arte-
riosus. In the fetus the ductus arteriosus con-
34. The correct answer is G. Cardiac troponin I nects the pulmonary trunk to the aorta and
levels (line A) become elevated in the first allows blood from the right ventricle to bypass
four hours after an MI and remain elevated for the lungs (which do not function at this time),
7-10 days. It is the most specific protein marker enter the aorta, and return to the umbilical ar-
for MI. However, creatine kinase (CK)-MB is teries. At birth, the increase in oxygen with the
the enzyme of choice for the detection of re- infant’s first breath results in decreased prosta-
infarction within the first week. If re-infarction glandin levels, which allows the ductus arterio-
occurs, CK-MB levels would again increase, sus to close. Closure is assisted by increased ox-
whereas troponin levels remain elevated from ygen stimulating the opening of the pulmonary
the previous event. vessels, which decreases vascular resistance,
thus leading to increased blood flow to the
Answer A is incorrect. AST (line C) is the
Cardiovascular
lungs. Failure of the ductus arteriosus to close
third enzyme to become elevated, as it gradu-
completely results in a PDA. This condition
ally increases over the first two days, then
is almost always present in premature infants
slowly declines. The serum level of this en-
with low surfactant production and low oxy-
zyme is not specific for cardiac cell damage.
gen levels. Indomethacin, a NSAID, inhibits
Answer B is incorrect. Brain natriuretic pep- prostaglandins and is frequently used to close a
tide becomes elevated as the atria are stretched PDA in neonates.
chronically as a result of volume overload, as
Answer A is incorrect. The first aortic arch
occurs in CHF. It is not represented in the im-
contributes part of the maxillary artery.
age, as it is not acutely elevated in MI.
Answer B is incorrect. The fourth aortic arch
Answer C is incorrect. C-reactive protein is a
gives rise to the aortic arch on the left and the
marker of inflammation. It has recently been
proximal right subclavian artery on the right.
shown in the JUPITER trial as possibly valu-
able as beginning statin use to lower cardiac Answer C is incorrect. The second aortic arch
risk in a person with a normal LDL cholesterol produces the stapedial artery and the hyoid ar-
level. It is not represented in the image, as it is tery.
not used to diagnose MI.
Answer E is incorrect. The third aortic arch
Answer D is incorrect. CK-MB (line B) levels gives rise to the common carotid artery and the
peak in the first 24 hours and then decrease. proximal part of the internal carotid artery.
190 Section II: Organ Systems • Answers
36. The correct answer is C. This patient is most the axilla usually involve pathology of the mi-
likely presenting with symptomatic aortic tral valve.
stenosis. One of the most common presenta-
Answer D is incorrect. An early diastolic de-
tions of this condition is new-onset syncope in
crescendo murmur radiating to the apex would
an older adult during an episode of exertion.
be associated with aortic insufficiency (also
This results from the inability to increase car-
known as aortic regurgitation). This condition
diac output during exertion due to a stenotic
can result in dyspnea, but generally is not as-
(usually calcified) valve. In addition, the ste-
sociated with syncope unless there is associ-
notic valve also causes a pressure build-up on
ated severe left ventricular dysfunction. Aortic
the left side of the heart, resulting in pulmo-
regurgitation will also present with a widened
nary congestion, as suggested by the bilateral
pulse pressure.
crackles in this patient. The classic heart mur-
High-Yield Systems
mur of aortic stenosis is a harsh crescendo- Answer E is incorrect. An early diastolic de-
decrescendo systolic murmur usually heard crescendo murmur could be associated with
best along the right upper sternal border, aortic insufficiency; however, the murmur of
which radiates to the carotids or the apex. aortic insufficiency would not radiate to the ax-
Other findings on cardiovascular examination illa. A murmur of this nature in the axilla may
that suggest aortic stenosis include a weak and be suggestive of mitral regurgitation; however,
delayed carotid pulse (pulsus parvus et tardus), this patient’s symptoms do not match with this
soft or absent A2 component of S2, displaced condition.
point of maximal impulse (PMI) with left ven- Answer F is incorrect. Mitral stenosis is associ-
tricular hypertrophy, and later left ventricular ated with an opening snap followed by a mid-
dysfunction (which results in a wide and dis- diastolic rumble. Mitral stenosis most com-
placed PMI). monly presents with dyspnea, but is generally
Answer A is incorrect. Mitral regurgitation is not associated with syncope, and does not pro-
associated with a blowing holosystolic murmur duce a weak and delayed carotid pulse. Mitral
best heard along the apex. When moderate to valve stenosis is often associated with a history
severe, this condition can result in dyspnea, of rheumatic fever.
Cardiovascular
posterior atrium, where the AV nodal cells are ease is the development of coronary artery an-
located. eurysms, which can result in MI. IV immuno-
globulin and aspirin is the appropriate therapy.
Answer A is incorrect. The diagonal branches
of the LAD are one or two large branches of Answer A is incorrect. Kawasaki disease is as-
the LAD that descend anteriorly across the sur- sociated with a high fever rather than with hy-
face of the left ventricle. A lesion here will not pothermia.
cause an inferior wall infarct of the heart.
Answer B is incorrect. Kawasaki disease is not
Answer B is incorrect. The left anterior de- associated with optic neuritis. This finding is
scending (LAD) branch of the left coronary more commonly associated with multiple scle-
artery supplies the anterior two-thirds of the rosis.
interventricular septum and the entire ante-
Answer C is incorrect. Kawasaki disease typi-
High-Yield Systems
rior wall of the left ventricle, as it travels along
cally does not involve the kidney, which is in
the anterior interventricular groove toward the
contrast to Henoch-Schönlein purpura, a vas-
apex of the heart. A lesion in the LAD would
culitic disease affecting mostly children.
result in an anterior wall MI. It does not sup-
ply the inferior wall of the heart, as described Answer D is incorrect. Kawasaki disease of-
in the vignette. A large proximal LAD infarc- ten occurs after an upper respiratory infection,
tion may also affect the conduction system, but it is not usually associated with pulmonary
but it typically affects the area below the AV symptoms.
node that courses through the interventricular
septum. This can sometimes result in bundle- 39. The correct answer is F. This woman is likely
branch blocks, higher-degree AV blocks, and suffering from prosthetic valve endocarditis.
even complete heart block. She may not have taken appropriate prophy-
lactic antibiotics before her root canal proce-
Answer C is incorrect. The left circumflex ar- dure, and her susceptible mitral valve after
tery is a branch of the LAD that travels toward rheumatic fever has been exposed to transient
the left side of the heart. A lesion here will bacteremia. Her symptoms, including low-
not cause an infarct on the inferior wall of the grade persistent fever, new-onset murmur,
Cardiovascular
heart. and insidious onset, suggest subacute bacterial
Answer D is incorrect. The left marginal ar- endocarditis. This is further supported by her
tery is a large branch of the left circumflex ar- physical examination, which reveals the pres-
tery. It continues on the left side of the heart ence of Roth spots (retinal hemorrhages), Os-
across its rounded obtuse margin. A lesion ler’s nodes (painful red nodules on digits), and
here does not cause an AV block or an inferior Janeway lesions (dark macules on palms and
wall infarct. soles). Given her clinical history and symp-
toms, the bacterium most likely to have caused
38. The correct answer is E. The child in this this episode is Streptococcus sanguis, part of
question likely has Kawasaki disease, a vascu- the viridans group. The most appropriate treat-
litis of unknown etiology that is hypothesized ment for such an infection is penicillin G.
to be an infectious or autoimmune response
Answer A is incorrect. Acyclovir is a guano-
(ie, molecular mimicry). The major symptoms
sine analogue antiviral drug used to treat her-
include a high fever of more than five days’
pes simplex and herpes zoster. This is a bacte-
duration, bilateral conjunctivitis, lip fissures,
rial endocarditis and not a viral infection.
strawberry tongue, palmar and plantar desqua-
mation, and cervical lymphadenopathy. Minor Answer B is incorrect. Caspofungin is an anti-
criteria may include elevated acute phase reac- fungal used to treat aspergillosis. It would not
tants, leukocytosis, thrombocytosis, and mild treat a gram-positive cocci infection.
elevation of liver function test results. The
most important complication of Kawasaki dis-
192 Section II: Organ Systems • Answers
Answer C is incorrect. Clindamycin, the treat- metic actions. It would thus not be indicated
ment for several important anaerobic infec- in this clinical scenario.
tions, works by blocking peptide bond forma-
Answer E is incorrect. Phenylephrine is an
tion at the 50S ribosomal subunit. This is not
a-adrenergic receptor agonist that causes vaso
an anaerobic infection.
constriction and rapid increases in total pe-
Answer D is incorrect. Mebendazole is an ripheral resistance. This dramatic increase in
antiparasitic drug used to treat roundworm afterload would be very dangerous for this pa-
infections such as pinworm and whipworm. tient.
Mebendazole is not used to treat bacterial en-
docarditis. 41. The correct answer is E. Nitrates are first-line
treatment in the management of angina. They
Answer E is incorrect. Metronidazole is a
act by stimulating the release of nitric oxide in
High-Yield Systems
ers cardiac output. The goal of therapy should heart rate and contractility to effectively re-
be restoring myocardial contractility and in- duce myocardial oxygen consumption. How-
creasing cardiac output. Glucagon acts as a ever, this is not the mechanism by which ni-
positive inotropic agent. It increases intracel- trates work in the treatment of angina.
lular cAMP levels independently of adrenergic
Answer C is incorrect. Nitrates act primarily
receptor signaling. Because its mechanism is
by stimulating venodilation to decrease preload
unaffected by adrenergic blockade, glucagon is
and thereby reduce myocardial oxygen con-
the drug of choice in b-blocker toxicity.
sumption. They do not act by increasing the
Answer A is incorrect. Albuterol is primarily oxygen supply to the myocardium.
a b2 agonist and has little positive inotropic ef-
Answer D is incorrect. Angina is caused by
fect.
atherosclerotic stenosis within coronary arter-
Answer B is incorrect. Cocaine causes vaso- ies that limit blood flow through those ves-
constriction and may lead to arrhythmia. It is sels. Coronary arterioles in patients with flow-
not indicated as treatment for b-blocker over- limiting coronary stenosis are already dilated
dose. to maintain resting blood flow. Therefore, any
vasodilating effects nitrates have on coronary
Answer C is incorrect. Although digitalis is
arteries are negligible in the setting of already
also a positive inotropic agent, it commonly
maximally dilated coronary arteries. Thus,
leads to atrioventricular block in the setting of
stimulating the vasodilation of coronary arteri-
bradycardia due to its strong parasympathomi-
Chapter 8: Cardiovascular • Answers 193
oles is not the primary mechanism of nitrates Answer E is incorrect. The afferent firing rate
in the treatment of angina. would decrease, not increase, with hypoten-
sion.
42. The correct answer is B. Isovolumetric con-
traction (phase II in the image) is the period 44. The correct answer is B. Dressler syndrome is
between mitral valve closing and aortic valve an autoimmune phenomenon that results in
opening. The cardiac musculature contracts fibrinous pericarditis. This delayed pericardi-
against the closed aortic valve to drastically el- tis typically develops 2-10 weeks post-MI and
evate ventricular pressure. presents clinically as chest pain and a pericar-
dial friction rub. It is generally treated with
Answer A is incorrect. Ventricular filling
nonsteroidal anti-inflammatory agents or corti-
(phase I in the image) is the period between
costeroids.
mitral valve opening and closing. Ventricu-
High-Yield Systems
lar pressure remains roughly equal to atrial Answer A is incorrect. Cardiac arrhythmia is a
pressure, as they are in direct communication common cause of post-MI death, typically oc-
while the mitral valve is open. curring the first few days following the event. It
is not associated with a friction rub.
Answer C is incorrect. Ventricular ejection
(phase III in the image) is the period between Answer C is incorrect. Left ventricular failure
aortic valve opening and closing. Volume falls occurs in 60% of people who suffer an MI and
precipitously as blood rushes into the aorta. can present as CHF, which can cause chest
pain, dyspnea, and an elevated jugular venous
Answer D is incorrect. Isovolumetric relax-
pressure. No friction rub is typically present.
ation (phase IV in the image) is the period in
which both the aortic and mitral valves are Answer D is incorrect. Because ischemic/
closed, thus keeping ventricular volume con- scarred myocardial tissue lacks normal contrac-
stant. This phase ends when the ventricular tility, there is increased blood stasis and forma-
pressure falls below the level of the atrial pres- tion of large mural thrombi. Smaller throm-
sure, and the mitral valve opens to allow fill- boemboli can break off these large mural
ing. thrombi and lead to cerebrovascular accidents,
Cardiovascular
transient ischemic attacks, and renal artery
43. The correct answer is C. The carotid sinus thrombosis. Post-MI arrhythmias are also a pro-
baroreceptor sends an afferent signal via the moter of blood stasis and subsequent thrombo-
glossopharyngeal nerve to the medulla, which embolic events. However, a friction rub does
in turn responds by increasing sympathetic not indicate thromboembolism.
outflow. This results in systemic vasoconstric-
Answer E is incorrect. Ventricular rupture is
tion, increased heart rate, increased contractil-
a serious cause of post-MI death that typically
ity, and increased blood pressure.
occurs 4-10 days after the initial event. It can
Answer A is incorrect. The baroreceptor lo- present with persistent chest pain, syncope,
cated in the aortic arch responds only to an in- and distended jugular veins, but most often
crease in blood pressure. it presents with sudden death. A friction rub
Answer B is incorrect. The correct efferent re- would not be observed.
sponse to a decreased baroreceptor afferent fir-
45. The correct answer is D. This patient experi-
ing rate would be increased sympathetic activ-
enced an aortic dissection, characterized by a
ity and decreased parasympathetic activity.
transverse tear through the intima and internal
Answer D is incorrect. The baroreceptor lo- media of the aortic wall. A blood-filled chan-
cated in the aortic arch responds only to an in- nel subsequently forms within the wall and is
crease in blood pressure. The parasympathetic at great risk of rupture, resulting in massive
nervous system is not activated to correct hypo- hemorrhage (the “pseudolumen” is the darker
tension. of the two lumens in the image). Hyperten-
194 Section II: Organ Systems • Answers
sion is the major risk factor contributing to the all automaticity of the conduction system and
damage of the blood vessel. Other risk factors would not result in randomly dropped QRS
include connective tissue diseases (eg, Ehlers- complexes.
Danlos and Marfan syndromes), pregnancy,
trauma, and aortic coarctation. 47. The correct answer is A. The patient died
suddenly a few hours after an acute MI. Fatal
Answer A is incorrect. Cocaine abuse can lead
arrhythmias are the most common cause of
to hypertension, which can later contribute to
death (also known as sudden cardiac death)
aortic dissection. Cocaine abuse would not be
in the first few hours of an infarction. Arrhyth-
the direct cause of the aortic dissection, how-
mias are due to disruption of the vascular sup-
ever.
ply to the conduction system, combined with
Answer B is incorrect. Diabetes is a risk factor myocardial irritability after injury. The patient
High-Yield Systems
for MI but not for aortic dissection. likely suffered from polymorphic ventricular
tachycardia or ventricular fibrillation. Also,
Answer C is incorrect. Hypercholesterolemia
ventricular tachyarrhythmias and intraventric-
contributes more significantly to atherosclero-
ular conduction abnormalities (such as a left
sis and subsequent MI than aortic dissection.
bundle branch block) are common long-term
Answer E is incorrect. Sickle cell disease is complications of an MI after the myocardium
not a direct cause of aortic dissection. becomes scar tissue and loses its intrinsic con-
ducting abilities.
46. The correct answer is B. The ECG findings
describe Mobitz type II second-degree heart Answer B is incorrect. Although less likely
block. A defect in the His-Purkinje system than a ventricular free-wall rupture, the in-
transmitting impulses from the atrioventricular terventricular septum also may rupture three-
(AV) node to the myocardium is usually, but seven days after infarction, leading to a VSD,
not always, responsible for this type of heart a left-to-right intracardiac shunt, and a low car-
block. diac output.
High-Yield Systems
This increased intracellular calcium level al-
ter the ischemic event. Papillary muscle rup-
lows greater amounts of calcium to be released
ture is not typically the underlying etiology
to the myofilaments during excitation, result-
of death in the acute setting. Rather, it could
ing in a positive inotropic effect. Increased
cause a low cardiac output and acute pulmo-
contractility of the heart increases stroke vol-
nary edema, likely requiring intubation until
ume, which in turn increases cardiac output.
surgery can be performed to repair the valve.
Glycosides are largely not used today because
of the advent of newer drugs that have fewer
48. The correct answer is A. The width of the P-V
adverse effects. The outstanding exception is
loop represents the stroke volume. Afterload
digoxin, which is still widely used to treat heart
refers to the aortic pressure against which the
failure and atrial fibrillation.
left ventricle pumps. Decreases in afterload
decrease the resistance against which the left Answer A is incorrect. b-Blockers inhibit sym-
ventricle must pump and, therefore, increase pathetic cardiac activation by blocking the ac-
the stroke volume of the cardiac cycle. Other tivity of catecholamines on the heart, decreas-
physiologic changes that increase the stroke ing heart rate and thus cardiac output. These
volume include increased preload and in- agents are specifically used in CHF in order
Cardiovascular
creased contractility. to decrease myocardial energy use and cardiac
oxygen demand.
Answer B is incorrect. Contractility describes
the intrinsic ability of the myocardium to Answer C is incorrect. Increased extracel-
pump against a given resistance. Decreased lular sodium levels in isolation would not in-
contractility in effect weakens the heart, reduc- crease cardiac output. Increasing cardiac out-
ing stroke volume. put would require an increase in intracellular
calcium levels, which is released from the sar-
Answer C is incorrect. Decreased preload is a
coplasmic reticulum. This may be achieved
reduction in the volume of blood that fills the
by the calcium influx triggered by sodium-
ventricle during diastole. Based on the Star-
calcium exchange channels. Thus increasing
ling relationship, in which force of contraction
intracellular sodium concentrations would in-
is proportional to the initial length of cardiac
crease contractility, but increased extracellular
muscle fibers, this decreased ventricular filling
sodium would not.
results in a reduction of stroke volume.
Answer D is incorrect. A decreased intracellu-
Answer D is incorrect. Increased arterial pres-
lar calcium level would decrease the contrac-
sure is synonymous with increased afterload.
tility of the heart, resulting in decreased stroke
Increased afterload results in decreased stroke
volume and thus decreased cardiac output.
volume based on an increase in pressure
against which the left ventricle must pump. Answer E is incorrect. Metabolic acidosis de-
creases contractility and stroke volume; thus
this would decrease cardiac output.
196 Section II: Organ Systems • Answers
50. The correct answer is D. Net fluid movement Answer B is incorrect. Decreased capillary
is governed by the equation JV = Kf[(Pc − Pi) pressure would decrease the pressure differen-
−(pc − pi)], where Kf is the filtration coefficient tial driving fluid into the interstitial space.
(factoring in membrane permeability), Pc is
Answer C is incorrect. Increased interstitial
capillary pressure, Pi is interstitial fluid pres-
fluid pressure would lead to greater resistance
sure, pc is capillary oncotic pressure, and pi is
to net fluid flow from the capillaries.
interstitial oncotic pressure. Increasing Pc, pi,
or the permeability of the capillaries will lead Answer E is incorrect. Increased plasma pro-
to a net flow of fluid from the capillaries into tein levels would cause an increase in capillary
the interstitium, leading to the edema that is oncotic pressure, leading to fluid retention in
seen in this patient. Likewise, decreasing pc the vascular space.
and Pi will also lead to net outward flow and
High-Yield Systems
edema.
Answer A is incorrect. Decreasing capil-
lary permeability would result in fluid being
trapped in the vascular space.
Cardiovascular
Chapter 17
Respiratory
451
452 Section II: Organ Systems • Questions
Q u e st i o n s
1. A 25-year-old woman with a history of asthma diffuse, bilateral interstitial infiltrates. Labo-
is brought to the emergency department by ratory studies are remarkable only for an el-
emergency medical services (EMS) after in- evated lactate dehydrogenase level. Which of
gesting a full bottle of theophylline in a suicide the following is the best choice for prophylaxis
attempt. At presentation, she is having a tonic- against this infection in a patient with a sulfa
clonic seizure. Her blood pressure is 80/40 allergy?
mmHg, respiratory rate is 30 breaths/minute,
(A) Aerosolized pentamidine
and her heart rate is 160/minute. The EMS
(B) Ciprofloxacin
personnel report that she has been seizing for
(C) Fluconazole
at least 15 minutes. What is the mechanism of
High-Yield Systems
(D) Terbinafine
action of the most appropriate drug to counter-
(E) Trimethoprim-sulfamethoxazole
act her intoxication?
(A) Decreasing intracellular cAMP 4. A previously healthy 41-year-old man misses
(B) Increasing intracellular cAMP through several days of work as a result of a viral illness
b2-adrenergic receptors with symptoms including fever, headache, and
(C) Increasing intracellular cAMP through fatigue. He also experiences a nonproductive
nonselective adrenergic receptors cough and a sore throat. By the third day, his
(D) Inhibiting the Na+-K+-2Cl- co-transporter symptoms begin to subside and he is able to
(E) Inhibiting the Na+-K+-ATPase pump return to work. The next week, however, he
experiences a rapid relapse. His cough returns,
2. A 28-year-old smoker presents to the emer- but now it is a productive cough with muco-
gency department because of sudden onset of purulent sputum. He also begins to experience
chest pain and dyspnea while at rest. His heart pleuritic chest pain. On visiting a physician,
rate is 115/min, respiratory rate is 24/min, and x-ray of the chest is ordered and is shown in
blood pressure is 140/80 mm Hg in both arms. the image. Which of the following describes
Lung examination shows decreased breath the structure of the viral genome that most
Respiratory
sounds and decreased fremitus on the right likely caused his initial illness?
with hyperresonance to percussion. Which of
the following would most likely be seen on this
patient’s x-ray of the chest?
(A) A widened mediastinum
(B) Barrel chest and flattened diaphragm
(C) Consolidation in the right lower lobe
(D) Contralateral deviation of the trachea
(E) Tracheal deviation to the ipsilateral side;
elevated diaphragm on the right side
High-Yield Systems
and does not improve with time. At autopsy
(B) Epiglottitis is associated with rhinorrhea
a few days later, the lungs are wet and heavy
and conjunctivitis
with areas of atelectasis alternating with oc-
(C) Epiglottitis often leads to respiratory dis-
casional dilated alveoli or alveolar ducts.
tress
Intra-
alveolar hyaline membranes consisting
(D) Symptom onset is gradual in epiglottitis
of fibrin and cellular debris are also present.
(E) The barking cough of epiglottitis becomes
A patent ductus arteriosus and intraventricu-
inspiratory stridor
lar brain hemorrhage are also seen at autopsy.
(F) Throat swab in epiglottitis would reveal
Which of the following is the likely etiology of
parainfluenza virus
the baby’s condition?
(A) Deficiency of hepatic glucuronyl transfer- 8. A 25-year-old man presents with new-onset
ase hemoptysis for the past 12 hours. The patient,
(B) Dipalmitoyl phosphatidylcholine defi- who recently immigrated to the United States
ciency from Vietnam, has had fever and night sweats
(C) Full-term uncomplicated pregnancy on a daily basis for the past four years. He has
(D) Lecithin:sphingomyelin ratio in amniotic no other complaints and no past medical his-
fluid >1.5 tory. The patient is diagnosed with active tu-
Respiratory
(E) Maternal steroid abuse prior to delivery berculosis and sent home on an antimycobac-
terial regimen. One month later, the patient
6. A 25-year-old medical student presents to the returns with new complaints of joint pain,
clinic with a nonproductive cough, low-grade photosensitivity, and a facial rash. Liver func-
fever, and malaise of three weeks’ duration. A tion tests are found to be elevated. Which of
few friends in his study group have been feel- the following is the mechanism of action of the
ing the same way. Sputum cultures are nega- drug with the adverse effects described above?
tive. The patient denies exposure to farm ani-
(A) Disrupts the cell membrane’s osmotic
mals, travel, or HIV. The physician treats for
properties
an atypical pneumonia. Which of the follow-
(B) Inhibits arabinosyl transferases
ing methods could help identify the organism
(C) Inhibits DNA-dependent RNA polymerase
responsible for this most likely causative patho-
encoded by the rpo gene
gen?
(D) Inhibits folic acid synthesis
(A) Acid-fast stain (E) Inhibits synthesis of mycolic acids
(B) Cold agglutinin testing
454 Section II: Organ Systems • Questions
9. A 26-year-old man presents to the emergency healthy. His vaccinations are up-to-date, and
department with respiratory difficulty. Starting he takes no medications. On chest x-ray, which
at age of 15, he began to have periodic short- portion of the lung is most likely to appear ab-
ness of breath. Serial pulmonary function tests normal?
revealed gradually increasing total capacity
(A) Left lower lobe
and residual volume. His renal function labo-
(B) Left upper lobe
ratory results were normal, but his aspartate
(C) Lingula
aminotransferase and alanine aminotransferase
(D) Right lower lobe
levels were significantly elevated. He began re-
(E) Right upper lobe
quiring home oxygen earlier this year, but his
condition continued to worsen. He died during 11. A 57-year-old man presents to his primary care
this admission, and an autopsy was conducted. physician for a routine wellness check. He
High-Yield Systems
Examination of his lungs showed the pathol- denies any complaints. Social history is sig-
ogy seen in the image. This patient’s illness nificant for a 50-pack-year smoking history. On
was most likely caused by which of the follow- physical examination, his vital signs are within
ing? normal limits except for his blood pressure,
which is 170/95 mm Hg. On his previous visit,
his blood pressure was 155/90 mm Hg. Which
of the following antihypertensive agents is rela-
tively contraindicated in this patient?
(A) Acebutolol
(B) Atenolol
(C) Esmolol
(D) Metoprolol
(E) Nadolol
Reproduced, with permission, from USMLERx.com. zone 3. In an experiment the physiologist ap-
plies a small amount of positive pressure ven-
tilation while studying blood flow in different
(A) Decreased levels of a1-antitrypsin lung zones. Which of the following will be
(B) Decreased levels of elastase noted in this experiment assuming the subject
(C) Increased levels of anti-neutrophil cyto- is standing?
plasmic autoantibodies
(A) Blood flow in zone 3 will be driven by the
(D) Increased levels of copper
difference between alveolar and venous
(E) Increased levels of iron
pressures
10. A 3-year-old boy is brought to the hospital with (B) Blood flow will be reduced in zone 1
acute shortness of breath. He was playing with (C) Blood flow will increase in zone 2
marbles in the playground when his mother (D) Regional differences between blood flow
noticed him cough and become acutely short will not be as great as differences in venti-
of breath. Her attempt to dislodge the object lation
on site was unsuccessful, and he was brought (E) Whether supine or standing, blood flow
to the hospital. Prior to this incident he was will remain uneven throughout the lung
zones
Chapter 17: Respiratory • Questions 455
13. A 60-year-old white man comes to his physi- (A) Pseudostratified columnar
cian because of a productive cough of a few (B) Simple squamous
months’ duration. The patient reports having (C) Stratified columnar
three of these episodes over the past two years, (D) Stratified squamous
with each episode lasting approximately four (E) Transitional
months. Histological examination of the lung
reveals hypertrophy of mucus-secreting glands 15. The oxygen-hemoglobin dissociation curve
in the bronchioles, with a Reid index >50%. represents the percent saturation of hemoglo-
What other changes are likely to be noted on bin with oxygen as a function of the partial
biopsy? pressure of oxygen in the blood. This curve
is sigmoidal in shape due to the change in af-
(A) Alveolar fluid and hyaline membranes
finity of heme groups for additional oxygen
(B) An infiltration of eosinophils and CD4+
High-Yield Systems
molecules. Which of the following in an adult
and TH2 lymphocytes
would cause a shift in the curve so that it re-
(C) An infiltration of monocytes and CD8+
sembles that of a neonate?
lymphocytes
(D) Necrotizing vasculitis and granulomatous (A) Increased 2,3-diphosphoglycerate
eosinophilic tissue infiltration (B) Increased partial pressure of carbon diox-
(E) Noncaseating granulomas ide in the blood
(C) Increased pH
14. A 65-year-old man with an 80-pack-year history (D) Increased temperature
of smoking presents to his physician because of (E) Intense exercise
a cough and increasing dyspnea over the past
six weeks. X-ray of the chest shows a 2-cm mass 16. A 50-year-old woman complains of dark-
in the left lower lobe of the lung, which is bi- colored urine and says she has not been feel-
opsied, revealing squamous cell carcinoma. A ing well for the past two-three weeks; she has
sample of non-neoplastic tissue from the lung generalized malaise and a nagging cough that
biopsy is shown in the image. Which of the occasionally is productive of blood-tinged
following types of epithelium not normally sputum. However, she noticed changes in
present in the lung lines the bronchus shown her urine for the first time today. Physical ex-
Respiratory
in this image? amination reveals an ill-appearing middle-aged
woman with a blood pressure of 180/110 mm
Hg. Diminished air entry in the lungs bilater-
ally, and an ulcerated lesion of the mucosa of
the right naris, are noted. There is no history
of asthma or allergies. Urinalysis is grossly posi-
tive for blood, and serum chemistry panel re-
veals a creatinine level of 1.7 mg/dL. What ad-
ditional finding would confirm the most likely
diagnosis?
(A) Eosinophilia on WBC differential
(B) IgA deposition in glomerular mesangium
(C) Linear IgG deposition in the kidney
(D) Positive for cytoplasmic anti-neutrophilic
cytoplasmic antibodies
(E) Positive for hepatitis B
Courtesy of Wikipedia.
456 Section II: Organ Systems • Questions
17. A 35-year-old African-American woman pre 19. A patient undergoing lung transplantation be-
sents to the clinic complaining of fatigue, cause of pulmonary fibrosis had his pressure-
dry cough, and dyspnea. X-ray of the chest is volume curves monitored throughout the
shown in the image. Lung parenchymal biopsy operation. The following events took place
reveals a noncaseating granuloma. Which of in the operating room: (1) his right lung was
the following cutaneous manifestations is asso- resected, (2) the new right lung was trans-
ciated with this condition? planted, and (3) positive end-expiratory pres-
sure (PEEP) was added to prevent pulmonary
edema. Which pressure-volume loop most
likely represents the patient’s pulmonary func-
tion when the patient was being ventilated on
one lung prior to the new lung being trans-
High-Yield Systems
planted?
D
A
600
Volume (mL)
C
B
(A) Loop A
18. An 18-year-old man comes to the physician (B) Loop B
complaining of a runny nose, sneezing, and (C) Loop C
difficulty breathing for the past two days. He (D) Loop D
says that when he goes outside or is away from
his house, his symptoms improve. On physical 20. A 5-year-old girl visiting from Mexico is
examination his turbinates are boggy and viola- brought to the emergency department by her
ceous. This type of reaction is most similar to aunt because of a sore throat and general mal-
which of the following? aise for the past three days. Physical examina-
tion reveals temperature of 38°C (100.4°F)
(A) Anaphylaxis
and a grayish-white membrane on the pharynx
(B) Contact dermatitis
that bleeds on attempted dislodgement. Which
(C) Goodpasture syndrome
of the following is the most appropriate culture
(D) Graft-versus-host disease
media for diagnosing this patient’s infection?
(E) Post-streptococcal glomerulonephritis
(A) Bordet-Gengou agar
(B) Chocolate agar with factors V and X
(C) Sabouraud agar
(D) Tellurite agar
(E) Thayer-Martin agar
Chapter 17: Respiratory • Questions 457
High-Yield Systems
bated and treated with steroids. Which of the
(C) Pyrazinamide toxicity
following is a characteristic of the alveoli of
(D) Rifampin toxicity
this neonate’s lungs?
(E) Tuberculous eye infection
(A) An increased pressure is needed to col-
22. A 36-year-old woman with a history of leuke- lapse the lungs
mia receives a bone marrow transplant. Two (B) Increased surface tension
and a half weeks later, she experiences fever, (C) The lungs have increased compliance
cough, and dyspnea. Bronchoalveolar lavage (D) The lungs lack a substance produced
reveals large cells with prominent intranuclear mainly by type I pneumocytes
inclusions, as shown in the image. What is the (E) The lungs produce a substance with a
most likely cause of this patient’s infection? lecithin:sphingomyelin ratio >1.5
Respiratory
constant urination and weight loss in the two
weeks prior to presentation. Laboratory tests
show a glucose level of 610 mg/dL, sodium
of 130 mEq/L, bicarbonate of 9 mEq/L, and
chloride of 95 mEq/L. Which of the following
would most likely be associated with this pa-
tient’s condition at presentation?
(A) Calcium oxalate crystals in the urine
(B) Decreased anion gap
Reproduced, with permission, from USMLERx.com. (C) Decreased blood partial pressure of carbon
dioxide
(D) Elevated blood partial pressure of carbon
(A) Candida albicans
dioxide
(B) Cytomegalovirus
(E) Hypokalemia
(C) Mycobacterium tuberculosis
458 Section II: Organ Systems • Questions
25. A medical student is asked to perform a car- 26. A 46-year-old woman presents to the emer-
diovascular examination on a patient. After 10 gency department because of a one-week his-
minutes of auscultation with no success, the tory of worsening nausea and lethargy. While
medical student gives up and asks his resident she is waiting to see the doctor she experiences
for help. The resident puts up the patient’s an- a seizure. Her past medical history is signifi-
teroposterior chest radiograph (see image) and cant for tuberculosis. Laboratory values show:
begins to explain the molecular etiology of this
Serum Na+: 109 mEq/L
patient’s condition. What is one respiratory
Serum osmolality: 255 mOsm/kg
complication that this patient is at increased
Urine osmolality: 850 mOsm/kg
risk of developing as a result of his condition?
Hematocrit: 27%
Which of the following drugs is also known to
High-Yield Systems
(A) Acute respiratory distress syndrome (A) Her intrapleural pressure is equal to atmo-
(B) Bronchiectasis sphere pressure during inspiration
(C) Carcinoma of the lung (B) Her intrapleural pressure is less than atmo-
(D) Exercise-induced asthma spheric pressure during inspiration
(E) Idiopathic pulmonary fibrosis (C) Pain from the trauma has made it difficult
to breathe
(D) Pressure within the pericardial space is in-
creased relative to the pleural space
(E) The elastic force of the chest wall is pull-
ing it inward
Chapter 17: Respiratory • Questions 459
28. A 74-year-old retired shipyard laborer with a (A) Autoimmune attack of lung parenchyma
45-pack-year smoking history and previous (B) Idiopathic (unknown) etiology
work in sandblasting and fiberglass operations (C) Living in a polluted city for years
presents with increasing shortness of breath (D) Long-term complication of steroid abuse
and peripheral edema. On physical exami- (E) Reactivation of a contained primary dis-
nation he is a thin, cyanotic man in moder- ease
ate pulmonary distress. His chest shows an (F) Working in a coal mine for 40 years
increased anteroposterior diameter, and the (G) Working in a shipyard for 40 years
breath sounds are faint with a prolonged expi-
ration. The liver edge is 3 cm below the right 30. A 158.8-kg (350-lb) man with a body mass
costal margin. There is no digital clubbing, index of 40 kg/m² comes to the physician
but marked peripheral edema is present. Arte- complaining of frequent fatigue, shortness of
High-Yield Systems
rial blood gas analysis reveals a partial oxygen breath, general sleepiness, and an inability to
pressure of 43 mm Hg, a partial carbon diox- concentrate. Physical examination shows an
ide pressure of 22 mm Hg, and a pH of 7.51. extremely obese, tired-looking man with hyper-
Which set of laboratory parameters is most tension and an elongated uvula. Which of the
likely to be found? following metabolic findings is most likely?
(A) Decreased serum glucose
Choice
Total (B) Increased HDL cholesterol
FEV1 FVC FEV1:FVC lung
capacity (C) Increased renal H+ reabsorption
A
(D) Increased renal HCO3- reabsorption
B (E) Increased renal HCO3- secretion
C normal
Respiratory
(A) A ventilation. X-ray of the chest demonstrates ho-
(B) B mogenous radiographic shadowing involving
(C) C both the lungs extensively. Culture of bron-
(D) D choalveolar lavage fluid on buffered charcoal
(E) E yeast extract demonstrates a coccobacillary
pathogen. What is the most likely causative or-
29. A 74-year-old patient presents with increased
ganism?
shortness of breath. A sputum sample reveals
golden-brown beaded fibers, which result (A)
Legionella pneumophila
from iron- and protein-coated fibers. On CT (B)
Listeria monocytogenes
scan, dense fibrocalcific plaques of the parietal (C)
Spirillum minus
pleura are seen. A particular pneumoconio- (D)
Staphylococcus aureus
sis is suspected. Which of the following is the (E)
Streptococcus pneumoniae
likely etiology of the patient’s condition?
460 Section II: Organ Systems • Questions
32. A 62-year-old woman underwent bilateral knee (B) Inhibition of phosphodiesterase resulting
replacement and was discharged without com- in increased cAMP levels
plications on postoperative day two. Nine days (C) Inhibition of the degranulation of mast
after surgery she develops severe respiratory cells
distress and dies suddenly in the emergency (D) Inhibition of the synthesis of cytokines
department. Postmortem examination of her (E) Stimulation of adenylyl cyclase resulting in
pulmonary artery reveals the pathology seen increased cAMP levels
in the image. What medical condition could
predispose to a similar pathology as observed in 34. In an attempt to better understand the patho-
this patient? physiology of obesity hypoventilation syn-
drome, a medical student is reviewing the ways
in which the body exerts control of respiration.
High-Yield Systems
High-Yield Systems
(E)
Streptococcus pneumoniae
Reproduced, with permission, from USMLERx.com.
38. A 26-year-old recent immigrant from Mexico
presents to the emergency department with a
(A) Surgery carries a risk of provoking para- three-week history of fevers accompanied by
neoplastic syndromes on removal of the night sweats and chills, weight loss of 2.3 kg
mass (5 lb), and cough that is often productive of
(B) Surgery has not been shown to improve blood-tinged sputum. Bronchoalveolar lavage
survival is performed and an acid-fast stain of the sam-
(C) Surgery is palliative but not curative ple reveals the organism shown in the image.
(D) Surgery often is curative in every lung can- Which of the following should be included in
cer without identifiable metastases this patient’s therapy to prevent a common tox-
(E) This lesion is likely to regress in seven icity of treatment?
years
Respiratory
without stopping to catch his breath. He has
also been plagued by a dry cough for the past
six months. He has not visited his primary
care physician because he is not a smoker and
does not believe that he could have a serious
pulmonary condition. He ignores his symp-
toms for another eight months, during which
time they continue to worsen. He finally visits
his physician at the urging of his wife. While
shaking hands, his physician notices that the
patient has clubbing of the fingers. A clini- Courtesy of Dr. George P. Kubica, Centers for Disease Con-
cal work-up and medical history fail to find a trol and Prevention.
cause for this restrictive lung disease. What is
the definitive therapy for this patient’s most
(A) Cobalamin
likely condition?
(B) Pyridoxine
(A) Albuterol (C) Vitamin B1
(B) Azathioprine (D) Vitamin C
(C) Cyclosporine (E) Vitamin E
(D) Lung transplantation
(E) Steroids
462 Section II: Organ Systems • Questions
High-Yield Systems
patient’s airways?
sistent cough she has had for the past three
months. The cough has been bothering her (A) Adult respiratory distress syndrome
a lot and making her anxious. She thinks the (B) Asthma
anxiety is why she has lost some weight re- (C) Atelectasis
cently. She also blames the anxiety whenever (D) Bronchiectasis
she wakes up in the middle of the night and (E) Churg-Strauss syndrome
finds herself drenched in sweat. Further his-
tory reveals she has rheumatoid arthritis (RA), 46. A 15-year-old boy with a history of severe
but her joint pains and swellings are well con- asthma presents to the emergency department
trolled by medications her rheumatologist has in obvious respiratory distress. After multiple
prescribed her. Following a physical examina- nebulizer treatments and doses of intravenous
tion, the physician orders an x-ray of the chest corticosteroids, he develops nausea, vomiting,
(see image). Based on the results, the physi- and weakness. Studies reveal a potassium level
cian immediately prescribes an antibiotic regi- of 2.6 mEq/L and U waves on ECG. Which of
men and asks her to discontinue one of the the following medications most likely would
drugs used to treat her RA. Which of the fol- have elicited these symptoms?
lowing drugs increased her risk of developing (A) Albuterol
Respiratory
the disease shown on the radiograph? (B) Cromolyn
(C) Ipratropium
(D) Theophylline
(E) Zileuton
47. A 32-year-old African-American woman pre (B) Gram-positive rods in chains with a pro-
sents to her physician complaining of a cough tein capsule
for the past two months and increased short- (C) Gram-negative pleomorphic coccobacilli
ness of breath over the past year. After com- requiring cysteine for growth
pleting a full physical examination, her physi- (D) Gram-positive weakly acid-fast rods form-
cian orders an x-ray of the chest, which shows ing long branching filaments
enlarged hilar nodes bilaterally as well as lung (E) Poorly stained gram-negative rods that
nodules. Results of a lung biopsy are shown in stain best on silver stain and require iron
the image. Which of the following treatments and cysteine.
would be most appropriate for this patient?
49. A 64-year-old man with sepsis has an increased
peripheral metabolic rate and hypercapnia.
High-Yield Systems
48. A 32-year-old man returns from an in-depth 50. A 65-year-old man with a 110-pack-year his-
tour of a sheep and goat farm. Five days later tory of smoking presents to his primary care
he develops fever, malaise, a dry cough, and physician because of shortness of breath, dys-
pressure in his chest. These symptoms re- pnea on exertion, and cough for three months’
solve after a few days. He then develops high duration. X-ray of the chest reveals flattened
fever, severe shortness of breath, chest pain, diaphragms bilaterally. The doctor orders pul-
cyanosis, and diaphoresis and is rushed to the monary function tests to evaluate the patient.
emergency department, where work-up reveals Which of the following pulmonary function
hemorrhagic mediastinitis, bloody pleural effu- test results would most likely be found in this
sions, and mediastinal widening on x-ray of the patient?
chest. Within a few hours the patient develops (A) Decreased FEV1:FVC
septic shock and dies. Which of the following (B) Decreased functional residual capacity
characterizes the most likely causative organ- (C) Decreased total lung capacity
ism? (D) Increased FEV1
(A) Gram-negative pleomorphic aerobic coc- (E) Increased FEV1:FVC
cobacilli
Chapter 17: Respiratory • Answers 465
An s w e r s
1. The correct answer is A. The drug theophyl- is treated by inserting a chest tube to remove
line is a phosphodiesterase inhibitor that leads the air from the pleural space.
to the decreased hydrolysis of cAMP to ad-
Answer A is incorrect. Aortic dissection, not a
enosine monophosphate. An overdose of the-
pneumothorax, would show up on x-ray of the
ophylline will therefore result in an elevated
chest as a widened mediastinum. Aortic dissec-
intracellular level of cAMP. b-blockers such as
tion can occur in trauma, or a dissecting aor-
metoprolol may therefore be given to reduce
tic aneurysm can occur in connective tissue
cAMP levels through inactivation of adenyl-
diseases such as Marfan disease. The physical
ate cyclase. A cardioselective b-blocker must
findings are not consistent with aortic dissec-
be used in cases of asthma to avoid inducing
High-Yield Systems
tion: first, the chest would not be hyperreso-
bronchial hyperreactivity.
nant to percussion and second, the blood pres-
Answer B is incorrect. b-agonists such as al sure would not be equal in both arms.
buterol would potentiate the effects of theoph-
Answer B is incorrect. Barrel chests and flat-
ylline by activating adenylate cyclase and in-
tened diaphragms are seen in patients with
creasing the conversion of ATP to cAMP.
obstructive lung diseases such as chronic em-
Answer C is incorrect. Epinephrine would physema. Although emphysema is caused
potentiate the effects of theophylline by acti- by smoking, this patient does not have a long
vating adenylate cyclase and increasing cAMP enough smoking history. It is possible that this
formation. man could have emphysema due to an a1-
antitrypsin deficiency, but in that case it would
Answer D is incorrect. Furosemide is a di-
not present this acutely.
uretic unrelated to theophylline overdose.
It works by inhibiting the Na+-K+-2Cl- co- Answer C is incorrect. Consolidation of the
transport system of the thick ascending loop of right lower lobe on x-ray of the chest usually
Henle. means pneumonia. The patient would present
with fever, productive cough, and a high WBC
Answer E is incorrect. Digoxin inhibits the
Respiratory
count. Physical exam would show decreased
Na+-K+-ATPase pump. It is used to increase
resonance on the affected side and increased
myocardial contractility in patients with con-
fremitus.
gestive heart failure. It is unrelated to theoph-
ylline overdose. Answer D is incorrect. In tension pneumotho-
rax, a flap-like pleural tear allows air to enter
2. The correct answer is E. This patient is most into the pleural cavity, but prevents its exit. It
likely suffering from a spontaneous pneumo- can be caused by penetrating trauma to the
thorax. Caused by the rupture of a small apical chest resulting in increased pleural cavity pres-
bleb on the surface of the lung, spontaneous sure. Clinical findings include sudden onset
pneumothoraces typically present in tall young of severe dyspnea, tympanitic percussion, and
men. The patient usually has sudden pain and absent breath sounds. There is tracheal devia-
dyspnea. Examination will show decreased tion and mediastinal structure deviation to the
breath sounds and hyperresonance on the af- contralateral side. If tension pneumothorax oc-
fected side. X-ray of the chest shows overexpan- curs on the left side, there would be compres-
sion of the rib cage and an elevated hemidia- sion of venous return to the heart. Treatment
phragm on the affected side. This paradoxical of tension pneumothorax is emergent needle
abdominal motion occurs because of the nega- decompression into the pleural cavity to re-
tive intrathoracic pressure that causes the fa- lieve the pressure.
tigued diaphragm to be pulled into the thorax
on the right side. Spontaneous pneumothorax
466 Section II: Organ Systems • Answers
3. The correct answer is A. This vignette sug- therefore, is, “Which viral illness predisposes
gests a patient with Pneumocystis jiroveci to subsequent bacterial pneumonia in an oth-
pneumonia based on the HIV status, physi- erwise healthy individual?” The classic an-
cal examination and x-ray findings, and the swer is influenza. The influenza virus is an
elevated lactate dehydrogenase level. Pro- enveloped, single-stranded RNA virus with a
phylactic therapy for P jiroveci pneumonia segmented genome that permits reassortment
is indicated for an HIV-positive patient with of the genes encoding the hemagglutinin and
a CD4+ T-lymphocyte count <200/μL. The neuraminidase proteins, resulting in the phe-
standard prophylactic therapy for P jiroveci nomenon of antigenic shift. Complications of
pneumonia is trimethoprim-sulfamethoxazole influenza include both viral pneumonia (due
(TMP-SMX). This combination, however, to a spreading of the illness into the lower res
is contraindicated for patients with a sulfa al- piratory tract) and bacterial pneumonia. The
High-Yield Systems
lergy, because sulfamethoxazole is a sulfa drug. latter is thought to be due largely to the fact
In these cases, the best alternative treatment is that influenza damages the epithelium of the
aerosolized pentamidine upper respiratory tract, compromising its abil-
ity to keep the lower respiratory tract sterile.
Answer B is incorrect. Ciprofloxacin is a fluo-
Streptococcus pneumoniae, Staphylococcus
roquinolone agent used to treat urinary tract
aureus, and Haemophilus influenzae are the
or gastrointestinal (GI) infections caused by
organisms most commonly seen in bacterial
gram-negative organisms. Adverse effects in-
pneumonia secondary to influenza. This chest
clude GI upset and tendinitis.
x-ray shows a consolidation in the right lower
Answer C is incorrect. Fluconazole is an anti- lobe along with a para-pneumonic effusion,
fungal agent used to treat systemic infections. highly suspicious for bacterial pneumonia.
It is also indicated for treatment of hypercorti-
Answer A is incorrect. The patient’s initial
solism found in ACTH-secreting tumors and
symptoms might be seen in a person with
polycystic ovarian syndrome. It inhibits hor-
acute infectious mononucleosis, consistent
mone synthesis.
with an infection with Epstein-Barr virus,
Answer D is incorrect. Terbinafine is an an- which is an enveloped, double-stranded DNA
tifungal agent that blocks ergosterol synthesis virus. However, the symptoms of mononucle-
Respiratory
by inhibiting squalene epoxidase. It is used to osis typically last longer than three days, and
treat dermatophytoses. bacterial pneumonia is not a common compli-
Answer E is incorrect. TMP-SMX is the ini- cation.
tial choice for prophylactic treatment of P jir Answer B is incorrect. The patient’s initial
oveci pneumonia unless patients are unable to symptoms are consistent with an upper re-
tolerate its harsh adverse effects, or if they have spiratory infection, which could be caused
a sulfa allergy. The most common adverse ef- by coronavirus, which is an enveloped, non-
fects are fever, rash, and bone marrow suppres- segmented, single-stranded RNA virus. Infec-
sion. tion with a coronavirus would not be expected
to lead to bacterial pneumonia in a healthy in-
4. The correct answer is C. From the history, dividual.
it appears that this man initially experienced
nonspecific viral symptoms, but there is not Answer D is incorrect. Non-enveloped, non-
enough information to determine which virus segmented, double-stranded RNA viruses in-
he has. What is clear, however, is that his ini- clude reovirus and rotavirus, neither of which
tial symptoms are distinct from what he expe- is a cause of respiratory illness in adults.
riences on relapse. The radiograph shows that Answer E is incorrect. Another cause of upper
he has lobar pneumonia (lower right lobe), respiratory infections in adults is rhinoviruses,
which can be caused by any number of bacte- which are non-enveloped, non-segmented,
rial species. The question that must be asked, single-stranded RNA viruses. Rhinovirus infec-
Chapter 17: Respiratory • Answers 467
tions are typically mild and uncomplicated in Answer C is incorrect. Gram stains are used to
healthy individuals, and a secondary bacterial visualize gram-positive or gram-negative bacte-
pneumonia would be atypical. ria. Mycoplasma pneumoniae is not visible on
Gram stain.
5. The correct answer is B. Dipalmitoyl phos-
Answer D is incorrect. India ink stain can be
phatidylcholine is a primary component of
used to visualize mucoid encapsulated yeasts
surfactant, which is deficient in neonatal res
such as Cryptococcus. This diagnostic tool is
piratory distress syndrome (NRDS). Chronic
not useful in identifying Mycoplasma species
hypoxemia to the fetus can result in congenital
or viruses.
abnormalities such as a patent ductus arterio-
sus and intraventricular brain hemorrhage. Answer E is incorrect. Serum polymerase
chain reaction (PCR) can be used to diagnose
Answer A is incorrect. Deficiency of hepatic
High-Yield Systems
viral illnesses such as HIV. PCR testing of the
glucuronyl transferase occurs in all newborns,
throat and sputum can be used to detect Myco
because the enzyme is not found at adult lev-
plasma, but this is not commonly performed.
els in neonates. This leads to physiologic jaun-
Cold agglutinin testing is preferred over PCR
dice, which has nothing to do with NRDS, but
for the detection of Mycoplasma.
could lead to kernicterus.
Answer C is incorrect. Predisposing factors for 7. The correct answer is C. Epiglottitis is a medi-
NRDS include prematurity, maternal diabetes cal emergency, and 90% of patients require
mellitus (DM), and birth by cesarean section. surgery to reestablish an airway. At presenta-
tion patients with epiglottitis can have little
Answer D is incorrect. In NRDS, the
or no respiratory compromise, but this can
lecithin:sphingomyelin ratio in the amniotic
progress to life-threatening respiratory distress
fluid is usually <1.5.
within a matter of hours.
Answer E is incorrect. Steroids are given to
Answer A is incorrect. Epiglottitis on x-ray
mothers who will deliver prematurely to try to
film of the neck reveals a “thumbs up” sign (ie,
prevent NRDS. Intratracheal administration of
“thumbprint” on radiograph), which correlates
artificial surfactant to the newborn can also be
with an inflamed epiglottis. Inflammation of
performed.
Respiratory
the larynx and sublgottic trachea is not associ-
6. The correct answer is B. In patients who ated with epiglottitis.
present with insidious onset of dry cough, Answer B is incorrect. Patients with epiglotti-
low-grade fever, headache, myalgias, nausea, tis do not have the symptoms or physical find-
or emesis, an atypical pneumonia should be ings of conjunctivitis or rhinorrhea. These
considered. Atypical pneumonias are mostly findings are more typical of croup. Epiglottitis
caused by Mycoplasma or viruses. Mycoplasma has additional symptoms of drooling and la-
cannot be cultured and is detected by the cold bored breathing.
agglutinin test, which measures the aggluti-
Answer D is incorrect. In general, the onset of
nation of immunoglobulins when they are
symptoms is abrupt with epiglottitis and grad-
cooled. X-ray of the chest is often more impres-
ual with croup.
sive than physical examination findings, and is
characterized by a patchy interstitial pattern. Answer E is incorrect. A typical barking cough
Treatment consists of antibiotic therapy with a is seen with croup, which may eventually lead
macrolide, usually azithromycin, for five days. to inspiratory stridor. Epiglottitis typically pre
sents with stridor and hoarseness.
Answer A is incorrect. The acid-fast stain is
used to diagnose mycobacterial illness, specifi- Answer F is incorrect. Most of the time, a throat
cally Mycobacterium tuberculosis. swab in epiglottitis will reveal Haemophilus
468 Section II: Organ Systems • Answers
mechanism of action of polymyxins. Polymyx- fect that impairs the transport of copper from
ins are not part of the treatment for TB. the liver into bile for excretion. The subse-
Answer B is incorrect. Ethambutol, not iso- quent accumulation of copper in the liver
niazid, inhibits the arabinosyl transferase- causes cirrhosis and leakage of copper into
mediated synthesis of arabinogalactin for the blood, where it damages other organs re-
mycobacterial cell walls. The side effects of sulting in neurologic, hematologic, and renal
ethambutol include dose-dependent visual dis- disease. Wilson disease is also characterized by
turbances, decreased visual acuity, red-green decreased serum ceruloplasmin due to a defect
color blindness, optic neuritis, and retinal in the incorporation of copper into ceruloplas-
damage. min. Wilson disease is not associated with lung
pathology.
Answer C is incorrect. The mechanism of ac-
tion here is that of rifampin, another antimyco- Answer E is incorrect. Excess iron is a sign of
bacterial drug. hereditary hemochromatosis, an autosomal-
recessive disorder characterized by excessive
Answer D is incorrect. Inhibition of folic acid dietary iron absorption due to the impaired
synthesis is the mechanism of action of dap- regulation of iron stores (HFE gene mutation).
sone.
Respiratory
more vertical and wider than the left, and as- Answer B is incorrect. Atenolol is cardioselec-
pirated particles are more likely to lodge at the tive and can be used in patients with asthma or
junction of the right inferior and right middle other obstructive lung diseases.
bronchi.
Answer C is incorrect. Esmolol is cardioselec-
Answer B is incorrect. The left main bron- tive and can be used in patients with asthma or
chus is narrower and less vertical than the right other obstructive lung diseases. However, it has
main bronchus. The right main bronchus is a very short half-life (about nine minutes), and
more vertical and wider than the left, and as- would not be used for long-term outpatient
pirated particles are more likely to lodge at the management.
junction of the right inferior and right middle
Answer D is incorrect. Metoprolol is cardio
bronchi.
selective and can be used in patients with
High-Yield Systems
Answer C is incorrect. The lingula is in the asthma or other obstructive lung diseases
left lung, and the left main bronchus is nar-
rower and less vertical than the right main 12. The correct answer is B. In zone 1, alveo-
bronchus. The right main bronchus is more lar pressure is greater than arterial pressure.
vertical and wider than the left, and aspirated There is risk of blood flow obstruction in
particles are more likely to lodge at the junc- zone 1 where ventilation/perfusion (V/Q) is
tion of the right inferior and right middle bron- high (wasted ventilation). In positive pressure
chi. ventilation alveolar pressure increases, which
also increases V/Q and compresses the capil-
Answer E is incorrect. When a person is su-
laries, limiting blood flow.
pine, aspiration pneumonia may affect the up-
per lobes and posterior segments of the lungs, Answer A is incorrect. Normally in zone 3,
since they become the gravity-dependent re- arterial pressure is greatest and alveolar pres-
gions when a person lies flat. sure is weakest. Thus the alveolar pressure is
too weak to impact blood flow, and the differ-
11. The correct answer is E. Nonselective b- ence between arterial and venous pressure de-
blockers are contraindicated in patients with termines blood flow. This will still be the case
lung disease because they can cause broncho- with a small amount of positive pressure venti-
Respiratory
constriction by blocking b2-receptors responsi- lation.
ble for relaxation of bronchial smooth muscle.
Answer C is incorrect. Normally in zone 2, ar-
Nadolol is a nonselective b-blocker and should
terial pressure is greater than alveolar pressure,
not be used in a patient with lung disease.
which is greater than venous pressure.
Other nonselective b-blockers include pro-
pranolol, timolol, and pindolol. Acebutolol, at- Answer D is incorrect. Regional differences
enolol, esmolol, metoprolol, and betaxolol are between blood flow are greater than differ-
cardioselective b1-blockers that should be fa- ences in ventilation, due to the effects of
vored in patients with lung/airway disease. Al- gravity. Positive pressure ventilation will not
though the stem does not specifically state that change this.
this patient has lung disease, smoking causes Answer E is incorrect. Blood flow changes de-
airway hyperreactivity and bronchoconstric- pending on whether a person is lying down or
tion. Adding a nonselective b-blocker could ex- standing up. When supine, blood flow is dis-
aggerate these adverse effects of smoking. tributed evenly throughout the lung because
Answer A is incorrect. Acebutolol is cardio the effects of gravity are not present.
selective and can be used in patients with
asthma or other obstructive lung diseases. 13. The correct answer is C. A diagnosis of
chronic bronchitis can be made based on the
patient’s symptoms and biopsy results. Along
with the hypertrophy of mucus-secreting
470 Section II: Organ Systems • Answers
glands and goblet cells, one typically sees an squamous epithelium is characterized by a
inflammatory infiltrate with a lymphocytic pre- single sheet of flattened cells lying on a basal
dominance, squamous cell metaplasia, and fi- lamina. It does not play a role in this case.
brosis.
Answer C is incorrect. Stratified columnar
Answer A is incorrect. Alveolar fluid and hya- epithelium is found in only a few places in the
line membranes are characteristic of adult res body, namely, the conjunctivae of the eye and
piratory distress syndrome. regions of the male urethra. It is composed of
a low polyhedral to cuboidal deeper layer in
Answer B is incorrect. Although these cells
contact with the basal lamina along with a su-
can be found in chronic bronchitis, they are
perficial layer of columnar cells.
much more typical of asthma.
Answer E is incorrect. The bladder is lined
Answer D is incorrect. Necrotizing vasculitis
High-Yield Systems
mouth, anus, vagina, and esophagus. While it Conversely, during exercise, temperature,
is wrongly believed that stratified epithelium is 2,3-DPG levels, and partial pressure of carbon
the result of the need for additional protection dioxide increase as a result of increased metab-
from the noxious smoke, the metaplasia actu- olism in the skeletal muscle. These would de-
ally results from genetic mutation related to crease hemoglobin affinity for oxygen, facilitat-
the developing cancer. ing unloading of oxygen to the tissue of highest
Answer A is incorrect. Pseudostratified colum- metabolic activity, and hence there would be
nar epithelium is the normal respiratory epi- a right shift of the oxygen-hemoglobin dissocia-
thelium on the right that is undergoing meta- tion curve.
plasia. This type of epithelium only appears Answer A is incorrect. An increase in 2,3-
stratified; however, all cells are in contact with DPG binds to deoxygenated hemoglobin in
basal lamina and only some cells reach the sur- RBCs, allosterically upregulating the ability
face of epithelium. of RBCs to release oxygen. Because it reduces
Answer B is incorrect. Simple squamous epi- RBC affinity for oxygen, increased 2,3-DPG
thelium lines alveoli, the loops of Henle, and right-shifts the oxygen-hemoglobin dissociation
blood vessels. Simple epithelium indicates that curve.
the epithelial membrane is composed of a sin- Answer B is incorrect. An increase in the par-
gle layer of cells, which helps when diffusion tial pressure of carbon dioxide in the blood
is important. Under the microscope, simple right-shift the curve, allowing increased deliv-
Chapter 17: Respiratory • Answers 471
ery of oxygen to tissues because of decreased immune response. Like Wegener granuloma
affinity for oxygen by RBCs. tosis, Goodpasture syndrome is associated
with hemorrhagic pneumonitis and glomeru-
Answer D is incorrect. Increased temperature
lonephritis. However, neither lesions of the
is an indicator of metabolic activity (and there-
nares and sinuses, nor positive c-ANCA titers,
fore increased oxygen demand). At high tem-
are characteristic of this disease. Moreover,
peratures, the curve shifts to the right, indicat-
Goodpasture syndrome more commonly af-
ing a decreased affinity for oxygen by RBCs.
fects young adults.
Answer E is incorrect. The acidosis that re-
Answer E is incorrect. Polyarteritis nodosa is
sults from the production of lactate during in-
an immune complex inflammation occurring
tense exercise causes a shift in the curve to the
in medium-sized vessels. It is associated with
right, reducing the affinity of RBCs for oxygen,
hepatitis B virus in 30% of patients, is associ-
High-Yield Systems
thereby allowing them to deliver more oxygen
ated with lesions of various ages, and can oc-
to tissues.
cur in almost any organ. However, it rarely
16. The correct answer is D. This patient’s con- involves the lung. Patients typically have fever,
stellation of symptoms is most consistent with weight loss, malaise, abdominal pain, melena,
Wegener granulomatosis, with the triad of fo- and hypertension, as well as cutaneous erup-
cal necrotizing vasculitis, necrotizing granu- tions, neurologic dysfunction, and hematuria.
lomas of the upper and/or lower airways, and
17. The correct answer is D. The patient has signs
necrotizing glomerulonephritis. Most patients
and symptoms of sarcoidosis, with classic race,
have positive titers for anti-neutrophil cytoplas-
pathology (noncaseating granuloma), and x-ray
mic antibodies with a cytosolic staining pattern
of the chest revealing prominent bilateral hi-
(c-ANCA). The disease is caused by systemic
lar lymphadenopathy, which is present in
granulomatous inflammation, particularly of
>90% of patients with sarcoidosis. Erythema
small- and medium-sized arteries such as those
nodosum, an inflammatory panniculitis, is
supplying the kidneys and lungs. If not treated
the most common cutaneous manifestation
with immunomodulating drugs, focal glomer-
of sarcoidosis, and frequently presents as bilat-
ulonephritis can progress to a crescentic form,
eral tender red bumps on the shins. Additional
with ensuing renal failure.
Respiratory
features of sarcoidosis include hypercalcemia
Answer A is incorrect. Eosinophilia is associ- due to increased activation of vitamin D by ac-
ated with Churg-Strauss syndrome, also known tivated macrophages.
as allergic granulomatous angiitis. Patients of-
Answer A is incorrect. Erythema infectiosum,
ten have asthma and/or allergies. Involvement
or fifth disease, is a common childhood viral
can include the lungs, heart, skin, kidneys, and
infection caused by erythrovirus or parvovi-
nerves. The lack of a history of asthma or aller-
rus B19. It is often associated with bright red
gies argues against Churg-Strauss syndrome.
cheeks in the early stages. It is not seen in sar-
Answer B is incorrect. IgA nephropathy (such coidosis.
as Berger disease), characterized by deposition
Answer B is incorrect. Erythema migrans is a
of IgA in glomerular mesangium, is a highly
characteristic rash that is often seen in patients
variable entity, ranging from asymptomatic
with early-stage Lyme disease. It often presents
hematuria to rapidly progressive glomerulone-
as a “bull’s-eye” lesion. It is not seen in sarcoi
phritis. IgA nephropathy is limited to the kid-
dosis.
neys.
Answer C is incorrect. Erythema multiforme
Answer C is incorrect. Goodpasture syn-
is a skin condition caused by inflammation of
drome is associated with anti-basement mem-
the microvasculature and mucous membranes.
brane IgG antibodies that recognize an epitope
It is most often precipitated by herpes sim-
on collagen IV. It is a type II hypersensitivity
plex virus infection, mycoplasma infection, or
472 Section II: Organ Systems • Answers
adverse drug reactions. It is not seen in sarcoi hypersensitivity reaction in which antigen-
dosis. antibody complexes activate complement. The
complex then causes the release of lysosomal
Answer E is incorrect. Erythema toxicum is
enzymes from neutrophils. Other type III reac-
a harmless rash that appears in approximately
tions include serum sickness, the Arthus-type
half of all newborns. It is not seen in sarcoi
reaction, and systemic lupus erythematosus
dosis.
(SLE).
18. The correct answer is A. This patient’s symp-
19. The correct answer is B. Loop B is notable for
toms are characteristic of allergic rhinitis,
the decreased volume and increased pressure
likely due to an indoor allergen. This response
of respiration compared to loops A, C, and D.
is an example of a type I hypersensitivity reac-
This loop was recorded while the patient was
tion in which an allergen cross-links antigen-
High-Yield Systems
ynx or tonsils; this pseudomembrane should ditional coverage of pyrazinamide during the
not be disrupted in order to avoid increased first two months.
absorption of the lethal exotoxin. Fever is usu-
Answer B is incorrect. Although rifampin is
ally mild or absent. It is seen very rarely in vac-
considered the best antituberculous agent, iso-
cinated populations but is endemic to certain
niazid is used for prophylaxis in asymptomatic
parts of the world. Culture of C diphtheriae
patients with a positive PPD test. A six-month
requires tellurite agar (Loeffler medium) to
course of isoniazid prevents activation of latent
prevent growth of normal upper respiratory
TB in 90% of patients for at least 20 years. Iso-
tract flora. Colonies will become gray to black
niazid blocks mycolic acid cell-wall synthesis
within 24 hours.
and is bactericidal for rapidly multiplying or-
Answer A is incorrect. Bordet-Gengou agar is ganisms. Major adverse effects include hepato-
used to culture Bordetella pertussis. Pertussis toxicity and peripheral neuropathy, but many
High-Yield Systems
presents with paroxysmal coughing spells and other adverse effects occur, such as lupus-like
whooping sounds on inspiration. syndrome and optic atrophy.
Answer B is incorrect. Chocolate agar is used Answer C is incorrect. Like isoniazid, the
to grow Haemophilus influenzae. Encapsulated spectrum of action of pyrazinamide is limited
strains of H influenzae cause invasive diseases to Mycobacterium tuberculosis. The site of ac-
such as septicemia, meningitis, cellulitis, sep- tivity for pyrazinamide is thought to be a fatty
tic arthritis, epiglottitis, and pneumonia. Non- acid synthase gene. The major adverse effect of
encapsulated strains are likely to cause otitis pyrazinamide therapy is hepatotoxicity, but it is
media, conjunctivitis, bronchitis, and sinusitis. rare at recommended dosages. Another major
adverse effect is hyperuricemia and subsequent
Answer C is incorrect. Sabouraud agar is used
gout.
to grow fungi.
Answer D is incorrect. Rifampin is the most
Answer E is incorrect. Thayer-Martin agar is
potent antituberculous agent available. Rif
a chocolate agar plate, which has VCN antibi-
ampin blocks DNA-dependent RNA poly-
otics (vancomycin, colistin, and nystatin) that
merase, preventing RNA synthesis. Although
suppress the growth of endogenous flora while
it is a better agent than isoniazid for prevent-
supporting Neisseria gonorrhoeae growth. This
Respiratory
ing active TB infection, it has a significant risk
patient does not have symptoms of gonorrhea.
of liver toxicity that outweighs its benefits as a
21. The correct answer is A. Ethambutol is ac- preventive medicine.
tive against Mycobacterium tuberculosis, and it Answer E is incorrect. Mycobacterium tuber
is among the first-line agents used to treat tu- culosis can have extrapulmonary manifesta-
berculosis (TB) infection (others are isoniazid, tions, but the eye is not commonly involved.
rifampin, and pyrazinamide). Ethambutol’s Miliary TB infection can affect the eye and
mechanism of action appears to be the inhibi- cause chorioretinitis, uveitis, and conjuncti-
tion of polymerization of cell-wall precursors. vitis, but these manifestations are rare. Color
Although the drug generally is well tolerated, blindness would not be associated with such
its most common adverse effects involve ocu- an infection.
lar toxicity such as loss of visual acuity and red-
green color blindness, which usually appears 22. The correct answer is B. Cytomegalovirus
several months after the initiation of treatment. (CMV) infection is a common complication
Ethambutol usually is used in an anti-TB in immunocompromised patients following
regimen with rifampin for patients who either bone-marrow transplantation. Histopathology
cannot tolerate isoniazid or are infected with shows large cells with intranuclear inclusions
isoniazid-resistant M tuberculosis. For chil- (so-called “owl’s eyes”) typical of CMV infec-
dren, most literature supports a regimen of six tion.
months with isoniazid and rifampin, with ad-
474 Section II: Organ Systems • Answers
Answer A is incorrect. Histopathology would anion gap (>10 mEq) as calculated by the fol-
show budding cells with pseudohyphae. lowing formula: Anion gap = Na+ - [HCO3- +
Cl-]. This leads to respiratory compensation
Answer C is incorrect. Histopathology would
by deep respiration (Kussmaul’s respiration),
show a granulomatous reaction, with the pos-
resulting in a decrease in blood partial pres-
sible presence of giant cells.
sure of carbon dioxide. The large anion gap is
Answer D is incorrect. Histopathology would due to the overproduction of ketones in the ab-
show acellular, foamy material. sence of insulin production.
Answer E is incorrect. Histopathology would Answer A is incorrect. Calcium oxalate crys-
show cysts with bradyzoites. tals may be seen in ethylene glycol poisoning,
which can be another cause of metabolic aci-
23. The correct answer is B. This child is suffer- dosis with an increased anion gap.
High-Yield Systems
bronchiectasis due to a nonfunctional muco- known cause of SIADH. HCTZ would be un-
ciliary elevator. likely to account for the degree of hyponatre-
mia (as well as the other laboratory abnormali-
Answer A is incorrect. Kartagener syndrome
ties) seen in this patient. Like indomethacin,
is not associated with acute respiratory distress
HCTZ can be used to treat nephrogenic DI.
syndrome.
Answer D is incorrect. Indomethacin has not
Answer C is incorrect. Kartagener syndrome
been associated with SIADH. It is, however,
is not associated with an increased incidence
sometimes used to treat nephrogenic DI. Pa-
of lung cancer.
tients with nephrogenic DI typically present
Answer D is incorrect. Kartagener syndrome with serum hyperosmolality, hypernatremia,
is not associated with exercise-induced asthma. and urine hypoosmolality.
Answer E is incorrect. Kartagener syndrome is
High-Yield Systems
Answer E is incorrect. Lithium toxicity has
not associated with idiopathic pulmonary fibro- been shown to cause nephrogenic DI rather
sis. than SIADH. In nephrogenic DI the kidneys
are unable to absorb water appropriately in re-
26. The correct answer is A. The patient is suffer- sponse to ADH. Patients present with produc-
ing from the syndrome of inappropriate ADH tion of large quantities of dilute urine, serum
secretion (SIADH), a condition in which ex- hyperosmolality, and hypernatremia. They
cessive ADH is secreted independently of se- report both polydipsia and polyuria. Medical
rum osmolality; this can be seen in a variety treatment of nephrogenic DI may consist of
of pulmonary diseases (including TB) as well hydrochlorothiazide or indomethacin.
as central nervous system (CNS) disorders that
enhance ADH release (eg, stroke, hemorrhage, 27. The correct answer is A. The patient’s pen-
infection, and trauma) and certain carcinomas etrating chest wound opened her intrapleural
(most commonly small cell lung carcinoma). space to the atmosphere. Therefore, as she at-
SIADH can also be an adverse effect of some tempts to inhale, her thoracic cavity expands
drugs, notably high-dose intravenous cyclo- but air enters through the wound, equalizing
phosphamide. Other drugs shown to cause the pressure; this prevents the normal expan-
SIADH include carbamazepine, vincristine, sion of the lungs. If air is not able to escape
Respiratory
vinblastine, cisplatin, amitriptyline, amio- through the wound duration exhalation, this is
darone, and monoamine oxidase inhibitors. called a tension pneumothorax, in which the
Excessive ADH secretion can lead to nausea, quantity of free air in the thoracic cavity in-
lethargy, seizures, and even coma. The pa- creases after each breath.
tient’s laboratory values are typical of someone
Answer B is incorrect. Intrapleural pressure
with SIADH, showing hyponatremia, serum
should be less than atmospheric pressure dur-
hypo-osmolality, urine hyperosmolarity, and
ing inspiration, allowing air entry. The prob-
decreased hematocrit secondary to dilution.
lem with this patient is that air is entering
Answer B is incorrect. Like lithium, demeclo- through a penetrating wound, rather than only
cycline has been associated with nephrogenic into the lungs.
diabetes insipidus (DI). Demeclocycline is
Answer C is incorrect. Pain with inspiration
not known to cause SIADH, the underlying
is a frequent complication of traumatic in-
disorder that accounts for this patient’s presen-
jury. While it may decrease the tidal volume,
tation. In fact, demeclocycline is sometimes
it would also increase breathing frequency,
used to treat SIADH.
resulting in an oxygen saturation closer to nor-
Answer C is incorrect. Hydrochlorothiazide mal. Additionally, this would not be corrected
(HCTZ) is a diuretic that may sometimes re- by chest tube placement.
sult in hyponatremia. However, this patient is
presenting with SIADH, and HCTZ is not a
476 Section II: Organ Systems • Answers
Answer D is incorrect. Increased pressure in 29. The correct answer is G. Working in a ship-
the pericardial space may result in a condition yard is associated with asbestos exposure.
called cardiac tamponade, during which the Chronic inhalation of asbestos fibers can result
heart cannot properly dilate. This may occur in asbestosis, which is marked histologically
when blood or other fluid enters the pericar- by ferruginous bodies that stain positively with
dial space. This can result in decreased blood Prussian blue. Asbestosis, unlike most other
pressure, increased jugular venous distention, pneumoconioses, results in marked predispo-
and distant heart sounds (called Beck’s triad). sition to bronchogenic carcinoma and to ma-
None of these clinical signs was noted in this lignant mesothelioma. Smoking and asbestos
patient; furthermore, a chest tube would not exposure together greatly increase one’s risk of
relieve the symptoms in cardiac tamponade. developing bronchogenic carcinoma.
Answer E is incorrect. The elastic properties Answer A is incorrect. Asbestosis is not related
High-Yield Systems
of the chest wall would tend to spring out, but to an autoimmune phenomenon.
the negative intrapleural pressure normally
Answer B is incorrect. The cause of asbesto-
created during inspiration opposes this ten-
sis is the inhalation of asbestos fibers into the
dency. After the penetrating injury equalized
lungs. Idiopathic restrictive lung diseases in-
the pressure of the intrapleural space and the
clude sarcoidosis and idiopathic pulmonary fi-
atmosphere, the chest wall will spring out, not
brosis.
pull inward.
Answer C is incorrect. Living in an urban area
28. The correct answer is B. Increased antero- for years can cause anthracosis, which is a re-
posterior diameter and prolonged expiration sult of inhalation of carbon dust. It is charac-
suggests the patient is suffering from chronic terized histologically by carbon-carrying mac-
obstructive pulmonary disease (COPD). Re- rophages and results in irregular black patches
duction of FEV1, forced vital capacity (FVC), visible on gross inspection. Anthracosis is
and the FEV1:FVC ratio are the hallmark of harmless.
airway obstruction. Total lung capacity (TLC)
Answer D is incorrect. Ferruginous bodies and
is increased due to lung hyperinflation, sec-
ivory-white pleural plaques are not long-term
ondary to expiratory flow limitation.
sequelae of steroid abuse.
Respiratory
uvula are very good indicators of OSA, as are (DVT) due to venous stasis. Virchow’s triad de-
his daytime sleepiness, inability to concentrate, scribes the three biggest risk factors for throm-
and hypertension. Periodic, recurrent asphyxia boembolism: (1) stasis, (2) hypercoagulability,
has the effect of causing a respiratory acidosis and (3) endothelial damage. To prevent DVTs
that, when present chronically, is compensated in postoperative, bed-bound patients, heparin
for by renal retention of HCO3-. is often started in the hospital and transitioned
to warfarin on an outpatient basis. Once in the
Answer A is incorrect. If anything, this pa-
lungs, DVTs can cause chest pain, shortness
tient’s glucose is likely elevated.
of breath, pulmonary hypertension, right heart
Answer B is incorrect. This patient likely has a strain/failure, or death. Her physical exam
decreased HDL level. findings are consistent with right heart strain
Answer C is incorrect. Increased reabsorption and are related to the obstruction of the pul-
High-Yield Systems
of H+ would worsen acidosis. monary artery, effectively forcing the right side
of heart to work against increased resistance
Answer E is incorrect. Renal secretion of (leading to delayed, pronounced pulmonic
HCO3- would worsen acidosis. valve closure and a split S2). Protein C defi-
ciency can cause a hereditary prothrombotic
31. The correct answer is A. The patient has a se- disorder because protein C is normally respon-
vere, potentially fatal, pneumonia with promi- sible for inactivating factors Va and VIIIa in
nent systemic symptoms. Culture on buffered the coagulation cascade.
charcoal yeast extract is the specific clue that
the organism is Legionella pneumophila. The Answer A is incorrect. The pathology shown
disease is respiratory legionellosis, also known indicates a vascular problem. It does not show
as legionnaire’s disease, so named because the signs of aspiration. Aspiration pneumonia is
disease was first described when it occurred in certainly a concern in elderly patients postop-
epidemic form after an American Legion con- eratively. Furthermore, aspiration pneumonia
vention at a Philadelphia hotel. Patients tend would likely lead to hospitalization first, rather
to be older and may have risk factors, includ- than sudden death caused by a pulmonary em-
ing cigarette use, alcoholism, diabetes, chronic bolus.
illness, or immunosuppressive therapy.
Respiratory
Answer B is incorrect. Factor V deficiency
Answer B is incorrect. Listeria monocytogenes does not increase the risk of coagulopathy.
causes listeriosis and is not a notable cause of However, factor V Leiden, a mutation in factor
pneumonia. V that confers resistance to activated protein
C, does result in a hypercoagulable state. Es-
Answer C is incorrect. Spirillum minus is sentially this mutation results in excessive fac-
a cause of rat-bite fever and is not a notable tor V activity. Do not confuse factor V Leiden
cause of pneumonia. with factor V deficiency.
Answer D is incorrect. Staphylococcus aureus Answer C is incorrect. Factor VIII deficiency
can cause pneumonia but is easily cultured on is a bleeding disorder. Congenital factor VIII
routine media. deficiency occurs in hemophilia A and is
Answer E is incorrect. Streptococcus pneu X-linked recessive. Acquired factor VIII defi-
moniae can cause pneumonia but is easily cul- ciency may occur in autoimmune conditions
tured on routine media. (inhibitors to clotting factors) or in liver disease
(defective production of factors). Factor VIII
32. The correct answer is D. This patient most deficiency is not associated with an increased
likely died from a massive pulmonary embo- risk of deep venous thrombosis or pulmonary
lism; the image shows an embolus in a pulmo- embolism.
nary artery. Immobilization postoperatively in- Answer E is incorrect. Thrombocytopenia,
creases a patient’s risk of deep vein thrombosis characterized by a decreased platelet count
478 Section II: Organ Systems • Answers
and increased bleeding time, would unlikely breathing. Stretch receptors are located in the
be the cause of a hypercoaguable state leading smooth muscle; irritant receptors are located
to a pulmonary embolus. in the airway epithelial cells; and the J (juxta-
capillary) receptors are located in the alveolar
33. The correct answer is A. Ipratropium bromide walls close to the capillaries.
is a muscarinic antagonist used to treat COPD
Answer E is incorrect. The aortic and carotid
and asthma. It competitively blocks muscarinic
bodies are not central chemoreceptors. They
receptors, preventing acetylcholine-mediated
are the peripheral chemoreceptors that are
bronchoconstriction. It is administered directly
able to respond to decreased partial pressure of
to the airway and is minimally absorbed, lead-
oxygen in arterial blood.
ing to few adverse events. At high doses, how-
ever, atropine-like toxicity may occur.
35. The correct answer is B. The patient has hy-
High-Yield Systems
action, which is false. There are few masses for causes of community-acquired pneumonia but
which nothing is done; an example is the cuta- is identified as the cause in only 3% of cases.
neous hemangioma seen in pediatric patients. The organism is present only in water sources
(eg, air conditioning systems, whirlpools, mist
36. The correct answer is D. Interstitial (or idio- sprayers) and causes infection when aerosol-
pathic) pulmonary fibrosis (IPF) is a chronic, ized water droplets are inhaled. Transmission
progressive fibrotic disorder of the lower res is not by person-to-person contact. Typically,
piratory tract that affects older adults. It is more severe illness is seen in patients who are
characterized by the abnormal proliferation >50 years of age, those who smoke, and those
of mesenchymal cells, disruption of collagen whose Gram stain shows neutrophils and very
structures, and impaired gas exchange. The few organisms, as in this case. Treatment is
exact pathogenesis of IPF is still unknown. If with erythromycin, because L pneumophila
High-Yield Systems
not treated, IPF often results in death within produces a b lactamase that renders it resistant
five years. Lung transplantation is currently to penicillin derivatives.
the only “cure” for this disease. On x-ray of
Answer A is incorrect. Bordetella pertussis
the chest, IPF usually is seen best in the lower
is a gram-negative rod that causes whooping
parts of both lungs as white lines in a netlike
cough, characterized by paroxysms of cough-
pattern.
ing followed by a loud inspiration, or “whoop.”
Answer A is incorrect. Albuterol is a b2- B pertussis infection does not typically cause
adrenergic agonist used in treating patients with pneumonia.
asthma. It is not a definitive therapy for IPF.
Answer B is incorrect. Haemophilus influen
Answer B is incorrect. Azathioprine may be zae type B is a gram-negative rod commonly
useful in the treatment of IPF, but a patient’s associated with acute epiglottitis or meningi-
pulmonary status will still deteriorate despite tis. It is an exclusively human pathogen that
its use. is transmitted by aerosolized droplets or direct
Answer C is incorrect. Cyclosporine is an im- contact with secretions. The H influenzae type
munosuppressive drug most commonly used B vaccine has rendered these infections far less
after transplantation. By inhibiting T-lympho- common, making it an unlikely agent in this
Respiratory
cytes from producing interleukin-2 and other scenario.
lymphokines, cyclosporine is able to reduce Answer D is incorrect. Mycobacterium tu
inflammation. Although it can be useful in the berculosis is an acid-fast mycobacterium that
treatment of IPF, a patient’s pulmonary status causes TB. The first exposure to M tubercu
often continues to deteriorate. losis, or primary TB, is usually asymptomatic.
Answer E is incorrect. Steroids may be useful Secondary, or reactivation, TB occurs after the
in the treatment of idiopathic pulmonary fibro- bacteria have been dormant for some time and
sis, but the patient’s pulmonary status generally re-emerge as a result of temporary weakening
continues to deteriorate despite their use. The of the immune system. This phase of disease
only cure for the disease is lung transplanta- is typically a chronic process characterized by
tion. low-grade fever, night sweats, weight loss, and a
productive cough. This chronic picture differ-
37. The correct answer is C. Legionella pneu entiates TB from the acute picture of Legion-
mophila is an aerobic, gram-negative rod that naires disease.
causes Legionnaire’s disease, a condition in Answer E is incorrect. Streptococcus pneu
which patients develop acute, severe pneumo- moniae presents acutely and is a significant
nia and a high fever. Other signs and symp- cause of bacterial pneumonia in adults. Un-
toms include hyponatremia (which is unique like L pneumophila, however, sputum culture
to this pneumonia) and CNS changes. Le- in S pneumoniae infection would reveal sig-
gionnaire’s disease is one of the most common nificant growth of gram-positive diplococci.
480 Section II: Organ Systems • Answers
sistance, isoniazid should never be used alone CT of the chest showing diffuse interstitial in-
to treat active TB. Isoniazid depletes pyridox- volvement. Measles-infected respiratory cells
ine (vitamin B6), which is required for the will fuse and form multinucleated giant cells,
production of dopamine, epinephrine, norepi- which can be detected in sputum samples.
nephrine, and monoamine neurotransmitters. Measles is a member of the Paramyxoviri-
Hence, one of the adverse effects of isoniazid dae family, a group of negative-sense, single-
therapy is peripheral neuropathy, which can stranded RNA viruses. In immunocompro-
be prevented by co-administration of vitamin mised hosts, measles pneumonia may evolve to
B6. giant cell pneumonia, which is often fatal.
Answer A is incorrect. There is no role for the Answer A is incorrect. Acid-fast bacilli would
administration of cobalamin (vitamin B12) in be expected in the sputum of a child infected
the treatment of TB. Vitamin B12 is used to with mycobacteria such as Mycobacterium tu
treat patients with deficiency who are showing berculosis.
neurologic symptoms and macrocytic anemia.
Answer B is incorrect. Cells with nuclei sur-
Vitamin B12 is a coenzyme that facilitates ar-
rounded by a halo and clear cytoplasm are
rangement of a hydrogen atom between two
koilocytes and would be found in cells infected
adjacent atoms and methyl group transfer be-
with human papillomavirus. This child has
Respiratory
lobar pneumonia. This child has other signs Answer A is incorrect. Choice A represents
and symptoms characteristic of measles. the inspiratory reserve volume, which is the
volume that can be inspired after inspiration of
40. The correct answer is A. This child suf- the tidal volume.
fers from a congenital diaphragmatic hernia
Answer B is incorrect. Choice B represents
caused by the failure of the diaphragm to prop-
the tidal volume, which is the volume inspired
erly form and close. The presence of bowel
or expired with each normal breath.
sounds in a lung zone indicates that abdomi-
nal contents have herniated past the boundary Answer C is incorrect. Choice C represents
of the diaphragm into the thorax. The devel- the expiratory reserve volume, which is the vol-
oping diaphragm is derived from the Septum ume that can be expired after the expiration of
transversum, Pleuroperitoneal folds, Body the tidal volume.
High-Yield Systems
wall, and Dorsal mesentery of the esophagus.
Answer E is incorrect. Choice E represents
These four components can be remembered
the inspiratory capacity, which is the sum of
by the mnemonic “Several Parts Build the Di-
tidal volume and inspiratory reserve volume.
aphragm.”
Answer F is incorrect. Choice F represents
Answer B is incorrect. A continuous cardiac
the functional reserve capacity. It is the sum of
murmur (ie, present during both systole and
the expiratory reserve volume and the residual
diastole) could be the consequence of a patent
volume, and it is the volume that remains in
ductus arteriosus, but is not related to the pleu-
the lungs after a tidal volume is expired.
roperitoneal folds and is unlikely to cause the
presentation in this patient. Answer G is incorrect. Choice G represents
vital capacity, which is the sum of tidal vol-
Answer C is incorrect. Marked splenomegaly
ume, inspiratory reserve volume, and expira-
in children has many etiologies, but is un-
tory reserve volume. Vital capacity (also called
likely to be consistent with the features of this
FVC) is the volume of air that can be forcibly
vignette. Causes of splenomegaly include
expired after a maximal inspiration.
congenital infections and metabolic genetic
disorders. Congenital infections include the 42. The correct answer is D. This patient is show-
ToRCHeS infections, which include Toxo-
Respiratory
ing signs of Cushing syndrome with a buffalo
plasmosis, Rubella, Cytomegalovirus, Herpes- hump and purple striae. Cushing syndrome
virus/HIV, and Syphilis. These infections often is caused by an excess of cortisol either be-
cause hepatosplenomegaly, jaundice, mental cause of a pituitary adenoma producing excess
retardation, and intrauterine growth retarda- ACTH, an adrenal adenoma producing ex-
tion. Lysosomal storage diseases such as Gau- cess cortisol, or ectopic ACTH production by
cher disease, Niemann-Pick disease, Hunter a neoplasm. This man’s smoking history and
syndrome, and Hurler syndrome also have lung nodule shown on chest radiography point
symptoms of hepatosplenomegaly. to lung cancer. Taken together with ectopic
Answer D is incorrect. Midline deviation of production of ACTH, this patient has para-
the trachea is commonly associated with pneu- neoplastic syndrome, with ectopic production
mothorax or space-occupying lesions of the of ACTH by the malignant lung mass. Of the
cervical region. different histological classifications of lung
cancer listed above, small cell carcinoma is
41. The correct answer is D. Choice D represents the most likely in the case for several reasons:
the residual volume, which is the volume that Squamous cell and small cell carcinomas are
remains in the lungs after a maximal expira- most closely linked to smoking history (>98%
tion. The residual volume increases dramati- are associated with smoking) and both pre
cally in emphysema. sent as central lesions such as that shown on
the x-ray film. Additionally, tumors producing
482 Section II: Organ Systems • Answers
ACTH or ADH are usually small cell carcino- x-ray of the chest as a collapsed lung with a
mas. mediastinum shifted away from the collapsed
lung. With pneumothorax, the patient should
Answer A is incorrect. Adenocarcinoma is the
be assessed for signs and symptoms of hemody-
most common lung cancer found in women
namic compromise. This patient, for example,
and nonsmokers (although 75% are found in
is hypotensive, tachycardic, and tachypnic,
smokers). Adenocarcinomas are usually pe-
and therefore requires urgent management.
ripherally located, and are less likely to cause
para-neoplastic conditions such as Cushing Answer A is incorrect. It is possible that this
syndrome. patient has a hemothorax, but this vignette de-
scribes a pneumothorax injury. A hemothorax
Answer B is incorrect. Bronchial carcinoid is
is characterized by blood in the thoracic cavity.
a rare neuroendocrine lung tumor that is not
High-Yield Systems
linked to smoking. These tumors cause cough, Answer B is incorrect. The stab wound is
hemoptysis, and an increase the number of res above the nipple, which is about the level of
piratory infections. Some of them are capable the fourth and fifth ribs, superior to the ninth
of producing serotonin and causes the classic and tenth ribs. It is possible that the man has
“carcinoid syndrome” characterized by epi- also sustained injury to his lower ribs, but this
sodic attacks of diarrhea, flushing, and cyano- would not be related to the knife injury and is
sis. not described in this vignette. Of note is the
risk that a fractured lower rib (11th or 12th)
Answer C is incorrect. Many cancers cause
may puncture the kidney, leading to retroperi-
metastases to the lung. These typically present
toneal bleeding.
as multiple discrete nodules found in all lobes.
The picture shown here is that of a single sol- Answer C is incorrect. A pleural effusion is
itary lesion, which is more likely to be a pri- seen on radiographs as a fluid collection in the
mary lung cancer. dependent portions of the thorax. Pleural effu-
sions can occur in heart failure, pneumonia,
Answer E is incorrect. Squamous cell carci-
or iatrogenic fluid overload (eg, improper fluid
noma accounts for 25%-40% of lung cancers
management of a hospitalized patient).
and is closely linked to smoking. Like small
cell carcinoma, squamous cell carcinoma also Answer E is incorrect. Right upper lobe con-
Respiratory
arises centrally and is associated with para- solidation would be consistent with right upper
neoplastic syndromes. However, while small lobe pneumonia, which is not described in this
cell carcinomas are responsible for Cushing vignette. One would expect to see a history of
syndrome and the SIADH secretion, squamous fever and other signs of infection, which is not
cell carcinomas usually cause hypercalcemia the case here. Also, radiographs would show an
by producing parathyroid hormone-related uninterrupted opacity.
peptide.
44. The correct answer is A. This patient is likely
43. The correct answer is D. This question re- suffering from a TB infection that was reacti-
quires knowledge of both the anatomy and the vated by her use of etanercept. The x-ray of the
physiology of the sucking chest wound, as de- chest shows a dense cavitary apical lung lesion
scribed in this patient. A penetrating wound that is highly indicative of a reactivated TB
to the chest can puncture the pleura, making infection. Etanercept is a fusion protein that
an opening for air to be sucked into the pleu- contains two identical tumor necrosis factor
ral space. With inspiration, the diaphragm (TNF)-receptor monomers fused to a human
descends, lowering the intrapleural pressure. IgG Fc domain. Therefore, it acts as a TNF
If there is a communication directly between antagonist. In TB infections, TNF (secreted
the pleural space and the outside world, air is by activated macrophages) recruits monocytes
sucked into this negative-pressure space and to form the epithelioid granulomas required to
collapses the lung. Pneumothorax is seen on contain the mycobacteria. When TNF is ef-
Chapter 17: Respiratory • Answers 483
fectively removed from the infection site (by Answer B is incorrect. Asthma is a condition
drugs or other forms of immunosuppression), associated with airway constriction, marked by
patients face an increased risk of reactivation wheezing. It does not present with hemoptysis
with caseation and cavitary lesions. or airway dilation
Answer B is incorrect. Methotrexate is an anti- Answer C is incorrect. Atelectasis is alveolar
inflamatory agent used in the treatment of RA. collapse and is not associated with airway dila-
It inhibits dihydrofolate reductase and blocks tion.
thymine synthesis. It is not an inhibitor of tu-
Answer E is incorrect. Churg-Strauss syn-
mor necrosis factor. The dosage of methotrex-
drome is a multisystem disease that commonly
ate used in the treatment of RA does not in-
affects the lung. It is characterized by eosino-
duce myelosuppression, although higher doses
philia and vasculitis, not airway dilation.
can produce this complication.
High-Yield Systems
Answer C is incorrect. Nonsteroidal anti- 46. The correct answer is A. b-agonists such as
inflammatory drugs (NSAIDs) do not impair albuterol may cause potassium to shift into
immunity, although they do impair platelet cells, resulting in hypokalemia. This may lead
function. These drugs can help decrease in- to ECG abnormalities due to destabilization of
flammation, but do not slow the progression of cardiac cell membranes, the classic examples
RA. It is quite unlikely that a patient with RA of which are U waves. Short-acting b-agonists
who complains of well-controlled joint pain such as albuterol are used in the treatment of
would be relying on NSAIDs alone. acute asthma exacerbations because of their
relaxing effects on bronchial smooth muscle.
Answer D is incorrect. Risedronate is a
Long-acting β-agonists such as salmeterol are
bisphosphonate used in the prevention and
used for prophylaxis of bronchospasm.
treatment of osteoporosis. Bisphosphonates are
not used in the treatment of RA, and do not Answer B is incorrect. Cromolyn inhibits
have immunosuppressive effects. antigen-induced bronchospasm by inhibiting
mediator release from bronchial mast cells,
Answer E is incorrect. Although the exact
and suppressing chemotaxis of neutrophils, eo-
mechanism of action of sulfasalazine is not
sinophils, and monocytes. It is used as a pro-
known, it is believed to suppress the activity
Respiratory
phylactic agent in mild to moderate asthma.
of natural killer cells and impair lymphocyte
Cromolyn is generally well tolerated, and ad-
transformation, which would not directly allow
verse effects are generally minor, including
mycobacteria to overcome immune surveil-
bronchospasm, cough, wheezing, angioedema,
lance and reactivate.
headache, and nausea.
45. The correct answer is D. Bronchiectasis can Answer C is incorrect. Ipratropium is an anti-
be caused by a chronic necrotizing infection muscarinic agent that is used for both asthma
of the bronchi leading to dilated airways. In ad- and COPD. Common adverse effects include
dition to bronchopulmonary infections, bron- cough, nausea, and dizziness. It is not known
chiectasis can be caused by bronchial obstruc- to cause hypokalemia.
tions or congenital abnormalities (bronchial
Answer D is incorrect. Theophylline most
cysts, tracheobronchial fistulas). Bronchiecta-
likely causes bronchodilation by increas-
sis is a common cause of hemoptysis and also
ing levels of cAMP. It does this by inhibiting
frequently presents with cough and dyspnea.
phosphodiesterase, an enzyme that hydrolyses
Answer A is incorrect. Adult respiratory dis- cAMP to AMP. Theophylline has a narrow
tress syndrome causes diffuse alveolar damage therapeutic window and may cause cardiotox-
that leads to increased alveolar capillary per- icity (and neurotoxicity) but does not result in
meability. It does not cause airway dilation. hypokalemia.
484 Section II: Organ Systems • Answers
ment of small cell lung cancer is operative, but patients suddenly develop acute respiratory dis-
in patients with stages IB and II disease, adju- tress and hypoxemia followed by hemorrhagic
vant therapy with cisplatin has shown a trend mediastinitis and bloody pleural effusions. A
toward improved survival. The image shows classic radiologic finding is mediastinal widen-
noncaseating granulomas, which are charac- ing. If a patient is not rapidly treated with peni-
teristic of sarcoidosis and would not be treated cillin, doxycycline, ciprofloxacin, or levofloxa-
with chemotherapy. cin, systemic infection can cause septic shock
(due to exotoxins produced by the bacteria)
Answer B is incorrect. Goodpasture syndrome
and death within 24 hours. Spores from sheep
is caused by anti-basement membrane anti-
or goat skin are the primary mode of trans-
bodies, which can be demonstrated on im-
mission in this kind of anthrax. Interestingly,
munofluorescence. It is not associated with
B anthracis is the only medically relevant bac-
noncaseating granulomas. Initial treatment of
teria with a protein capsule.
Goodpasture syndrome is a five-day course of
methylprednisolone followed by a long taper Answer A is incorrect. This describes Brucella.
and maintenance. However, if the disease is Brucella is transmitted from cattle to humans
particularly severe, immunosuppressive agents who have contact with infected animal meat,
such as cyclophosphamide or azathioprine milk products, or aborted animal placentas.
may be started. The pathogen penetrates multiple organs, in-
cluding the lungs, skin, conjunctiva, and GI
tract. Patients with brucellosis have systemic
Chapter 17: Respiratory • Answers 485
High-Yield Systems
bon dioxide is converted to bicarbonate, bicar-
very similar to inhalational anthrax; however,
bonate does not bind to hemoglobin; hydrogen
hemorrhagic mediastinitis is not seen in tu-
ions bind hemoglobin. Dissolved carbon di-
laremia, and death does not occur within 24
oxide that remains in the plasma accounts for
hours. Tularemia is also associated with rabbit,
about 5% of transport.
tick, or deerfly contact.
Answer D is incorrect. This describes Nocar 50. The correct answer is A. The long smoking
dia asteroides, which causes pulmonary infec- history and presence of flattened diaphragms
tions primarily in immunocompromised indi- on radiographs are suggestive of COPD. The
viduals. hallmark of obstructive lung disease is a de-
creased FEV1:FVC ratio (to <80%). FEV1
Answer E is incorrect. This describes Legion
is reduced in patients with COPD because
ella pneumophila. Legionella is a cause of
of obstruction. In restrictive lung diseases,
severe pneumonia, particularly in cigarette
FEV1:FVC is normal or increased.
smokers and immunocompromised individu-
als. It is associated with environmental water Answer B is incorrect. Functional residual ca-
sources. It does not cause mediastinitis or hem- pacity (FRC) is increased in COPD because
orrhagic pleural effusions. the patient is unable to expire air fully, result-
Respiratory
ing in air trapping in the lungs. In restrictive
49. The correct answer is C. In the tissues, more lung disease, FRC is decreased because the
carbon dioxide is being produced because of lung has restricted expansion so all lung vol-
the increased metabolic rate. The additional umes are decreased.
carbon dioxide enters the RBC and is com-
Answer C is incorrect. The total lung capacity
bined with water by carbonic anhydrase to
in patients with COPD increases. It decreases
form H2CO3, which then dissociates into hy-
in patients with restrictive lung disease.
drogen and bicarbonate. The hydrogen ions
are buffered by deoxyhemoglobin, while the Answer D is incorrect. COPD has a character-
bicarbonate diffuses out of the RBCs in ex- istically decreased FEV1.
change for chloride ions. This is called the Answer E is incorrect. Emphysema is an ob-
chloride shift. structive lung disease, which has a characteris-
Answer A is incorrect. Bicarbonate travels in tically decreased FEV1:FVC ratio. Restrictive
the plasma, not the RBCs. When it gets to the lung diseases can have increased FEV1:FVC
lung, it enters the RBCs, transforms back to ratios because FVC is reduced along with total
water and carbon dioxide, and the latter is ex- lung volume.
haled.
Answer B is incorrect. The primary transport
of carbon dioxide in the blood (90%) is via bi-